SlideShare una empresa de Scribd logo
1 de 31
ASIGNATURA ABASTECIMIENTOS DE AGUA Y ALCANTARILLADO - CIC404 HOJA Nº 01/04
PRE-DIMENSIONAMIENTO DE LA TOMA Y LÍNEA DE CONDUCCIÓN
1ero.
AÑO BASE DE OBSERVACIÓN ESTADÍSTISTICA-CATASTRAL: 2019 AÑO DE INICIO DE OPERACIONES: 2025
i)
CUADRO 01.-
DEMANDA CATASTRAL: Zona "A:GH" Zona "B:HI" Zona "C:IJ" Total
Poblac.total año 2019 ( hab. ) 26,500 17,750 8,750 53,000
Dotación 2019 (Lt./hab./día) 375.00 248.00 178.00 (Referencia:Catastro)
Consumo prom.2019 (Lt./seg.) 115.02 50.95 18.03 184.00
AA.HH.
PERIODO DE DISEÑO : 20 AÑOS (Dato)
AÑO FINAL DE OPERACIÓN: 2045
ii)
Zona "A:GH"
AÑO POBLACIÓN INC. Pob/year
2019 26,500 hab.
2024 27,438 hab. 188 100.00%
2029 28,151 hab. 143 -45 76.06%
2034 28,729 hab. 116 -27 81.12% Justificación inicial para construcción en 1 etapa
2039 29,216 hab. 97 -19 83.62%
2044 29,638 hab. 84 -13 86.60% -68.42% Población en etapa de saturación
2049 30,010 hab. 74 -10 88.10%
2054 30,344 hab. 67 -7 90.54%
Pob 2025 27,581 hab.
Pob 2030 28,267 hab.
Pob 2035 28,826 hab.
Pob 2040 29,300 hab.
Pob 2045 29,712 hab.
CRECIMIENTO TARDÍO CON ÍNDICE DECRECIENTE=
CRECIMIENTO LOGARÍTMICO
FACULTAD DE INGENIERIA CIVIL Y ARQUITECTURA
ESCUELA ACÁDEMICA PROFESIONAL DE INGENIERÍA CIVIL
CUESTIONARIO 17: Evaluación sumativa 02-AAA2020.2g1 (23/12/2020) - Peso 5
Ing. Civil Clifton Paucar y Montenegro - REG. CIP. 45773 - C. M Sc. Ing.Hidráulica - UNI
Para pre-dimensionar la capacidad requeridad de la toma y línea de conducción iniciamos con el cálculo del caudal de diseño en función a los datos suministrados.
Iniciamos el proceso de estimación elaborando el cuadro de datos y calculando el consumo promedio en función a la dotación promedio CATASTRAL O REGLAMENTARIA (según términos de
referencia, rentabilidad y tipo de proyecto).
Calculamos la población futura en funcIón al periodo de diseño y las ecuaciones generadas usando el Método de Incrementos cada 5 años en función a la población base de observación (2019)
hasta el año final del período de diseño que según cálculo económico efectuado es de 20 años, partiendo del año de inicio de operación (2025) + 20 años = Año 2045
Ecuación promedio representativa de tres o mas ecuaciones que definen el crecimiento poblacional (Pto
inical:-Pto Final): FINES ACADÉMICOS
PARA DETERMINAR LA ECUACIÓN CARACTERÍSTICA DEL TIPO DE CRECIMIENTO EN EL
QUE SE ENCUENTRA LA POBLACION TENEMOS QUE INTERPOLAR LOS DATOS
CENSALES A PERIODOS CONSTANTES DE 5AÑOS Y ENCONTRAREMOS LA CURVA CON
UN COEF. DE CORRELACIÓN LO MÁS CERCANO O IGUAL A 1, QUE ES LA QUE MEJOR
35,000
Xi
i e
Y 00033
.
0
22222 

Zona "B:HI"
AÑO POBLACIÓN INC. Pob/year
2019 17,750 hab.
2024 17,758 hab. 2 100.00%
2029 17,766 hab. 2 0 100.00%
2034 17,774 hab. 2 0 100.00% Justificación inicial para construcción en 1 etapa
2039 17,782 hab. 2 0 100.00%
2044 17,790 hab. 2 0 100.00% 0.00% Población en etapa de crecimiento
2049 17,798 hab. 2 0 100.00%
2054 17,806 hab. 2 0 100.00%
Pob 2025 17,760 hab.
Pob 2030 17,768 hab.
Pob 2035 17,776 hab.
Pob 2040 17,784 hab.
Pob 2045 17,792 hab.
CRECIMIENTO TARDÍO CON ÍNDICE DECRECIENTE=
CRECIMIENTO LOGARÍTMICO
AL TENER UN CRECIMIENTO ANUAL PORCENTUAL CONSTANTE DECRECIENTE
PERCÁPITE POSITIVO ES APLICABLE EN ESTE CASO LA EXPRESIÓN DE CAPEN
VALIDADA CON EVALUACIONES ADICIONALES SOCIO-ECONÓMICAS Y EXPANSIÓN
FÍSICA
PARA DETERMINAR LA ECUACIÓN CARACTERÍSTICA DEL TIPO DE CRECIMIENTO EN EL
QUE SE ENCUENTRA LA POBLACION TENEMOS QUE INTERPOLAR LOS DATOS
CENSALES A PERIODOS CONSTANTES DE 5AÑOS Y ENCONTRAREMOS LA CURVA CON
UN COEF. DE CORRELACIÓN LO MÁS CERCANO O IGUAL A 1QUE ES LA QUE MEJOR
REPRESENTA EN TEORIA EL CRECIMIENTO HISTÓRICO DE LA ZONA
OBSERVANDO LA CURVA TIENE LA ÚLTIMA ETAPA DE CRECIMIENTO: TARDÍO
(Saturación), CUYO TIPO DE CRECIMIENTO SE ENCUENTRA DIRECTAMENTE
INFLUENCIADO POR ASPECTOS DE TIPO ESPACIAL, SOCIAL Y ECONÓMICO.
Ecuación promedio representativa de tres o mas ecuaciones que definen el crecimiento poblacional (Pto
inical:-Pto Final): FINES ACADÉMICOS
PARA DETERMINAR LA ECUACIÓN CARACTERÍSTICA DEL TIPO DE CRECIMIENTO EN EL
QUE SE ENCUENTRA LA POBLACION TENEMOS QUE INTERPOLAR LOS DATOS
CENSALES A PERIODOS CONSTANTES DE 5AÑOS Y ENCONTRAREMOS LA CURVA CON
UN COEF. DE CORRELACIÓN LO MÁS CERCANO O IGUAL A 1, QUE ES LA QUE MEJOR
REPRESENTA EN TEORIA EL CRECIMIENTO HISTÓRICO DE LA ZONA
OBSERVANDO LA CURVA TIENE LA ÚLTIMA ETAPA DE CRECIMIENTO: TARDÍO
(Saturación), CUYO TIPO DE CRECIMIENTO SE ENCUENTRA DIRECTAMENTE
INFLUENCIADO POR ASPECTOS DE TIPO ESPACIAL, SOCIAL Y ECONÓMICO.
y = -46.048x2 + 946.76x + 25667
R² = 0.9985
0
5,000
10,000
15,000
20,000
25,000
30,000
35,000
2019 2024 2029 2034 2039 2044 2049 2054
POBLACIÓN INC. Pob/year Polinómica (POBLACIÓN)
y = 17742e0.0004x
R² = 1
8,000
10,000
12,000
14,000
16,000
18,000
20,000
Xi
i e
Y 00067
.
0
6677 

Rpta. A) Zona B
Zona "C:IJ"
AÑO POBLACIÓN INC. Pob/year
2019 8,750 hab.
2024 14,569 hab. 1,164 100.00%
2029 25,003 hab. 2,087 923 179.30%
2034 45,206 hab. 4,041 1,954 193.63% Justificación inicial para construcción en 1 etapa
2039 89,771 hab. 8,913 4,872 220.56%
2044 213,474 hab. 24,741 15,828 277.58% 324.88% Población en etapa de crecimiento geométrico
2049 740,023 hab. 105,310 80,569 425.65%
2054 5,994,336 hab. 1,050,863 945,553 997.88% Justificación inicial para construcción en varias etapas
Pob 2025 16,656 hab.
Pob 2030 29,044 hab.
Pob 2035 54,119 hab.
Pob 2040 114,512 hab.
Pob 2045 318,784 hab.
CRECIMIENTO TEMPRANO CON ÍNDICE CRECIENTE=
CRECIMIENTO GEOMÉTRICO
POR LO TANTO LA CURVA TIENE UN CRECIMIENTO GEOMETRICO CON
ECUACION EXPONENCIAL COMO SE MUESTRA EN EL GRAFICO.
AL TENER UN CRECIMIENTO ANUAL PORCENTUAL, POSITIVO ES APLICABLE
EN ESTE CASO ES EL QUE MEJORES RESULTADOS REPORTA EL USO DE LA
EXPRESIÓN DE CAPEN
OBSERVANDO LA CURVA TIENE LA ÚLTIMA ETAPA DE CRECIMIENTO: TARDÍO
(Saturación), CUYO TIPO DE CRECIMIENTO SE ENCUENTRA DIRECTAMENTE
INFLUENCIADO POR ASPECTOS DE TIPO ESPACIAL, SOCIAL Y ECONÓMICO.
AL TENER UN CRECIMIENTO ANUAL PORCENTUAL CONSTANTE NO ES APLICABLE EN
ESTE CASO LA EXPRESIÓN DE CAPEN DE ACUERDO A LAS CONSIDERACIONES
INCIALES DE DISEÑO(PARÁMETROS DE CONTORNO). DEBERÁ USARSE OTRA
ECUACIÓN BASADA EN ESTUDIOS SOCIO-ECONÓMICOS
PARA DETERMINAR LA ECUACIÓN CARACTERÍSTICA DEL TIPO DE
CRECIMIENTO EN EL QUE SE ENCUENTRA LA POBLACION TENEMOS QUE
INTERPOLAR LOS DATOS CENSALES A PERIODOS CONSTANTES DE 5 AÑOS
Y ENCONTRAREMOS LA CURVA CON UN COEF. DE CORRELACIÓN LO MÁS
CERCANO O IGUAL A 1 QUE ES UNA DE LAS QUE MEJOR REPRESENTA EN
TEORIA EL CRECIMIENTO HISTÓRICO DE LA ZONA
Ecuación promedio representativa de tres o mas ecuaciones que definen el crecimiento poblacional (Pto
inical:-Pto Final): FINES ACADÉMICOS
0
2,000
4,000
6,000
8,000
2019 2024 2029 2034 2039 2044 2049 2054
POBLACIÓN INC. Pob/year Exponencial (POBLACIÓN)
y = 2570.5x6 - 60081x5 + 554984x4 - 3E+06x3 + 6E+06x2 - 7E+06x + 3E+06
R² = 0.9999
-1,000,000
0
1,000,000
2,000,000
3,000,000
4,000,000
5,000,000
6,000,000
7,000,000
2019 2024 2029 2034 2039 2044 2049 2054
i
i X
Y 00088
.
0
8
.
58 

ETAPAS CONSTRUCTIVAS A CONSIDERAR (Verif. 1): 1 ETAPA
iii)
Se usará el cuadro de Relación: Diámetro versus Velocidad Económica ( Simón Arocha Ravelo - Normas INOS-Venezuela-1978)
ø(Pulg) 3 4 6 8 10 12 14
V máx (m/s) 0.70 0.75 0.80 0.90 1.00 1.10 1.20
Q ( lit/seg.) 3.05 5.89 14.14 28.27 49.09 77.75 115.45
ø(Pulg) 16 18 20 24 30
V máx (m/s) 1.25 1.30 1.40 1.60 1.60
Q ( lit/seg.) 157.1 206.76 274.9 452.39 729.6
DEMANDA CATASTRAL: Zona "A:GH" Zona "B:HI" Zona "C:IJ" Total Φecon.i Dotac. Correg. Zona B c / 5%
Pob.Total año 2019( Hab. ) 26,500 17,750 8,750 53,000
Dotación 2019(L/h/d) 375.00 248.00 178.00 248.00 248.00
Consumo prom.2019(L/s) 115.02 50.95 18.03 184.00 18"
Pob.Total año 2025(Hab. ) 27,581 17,760 16,656 61,997 17,760
Dotación Correg. 2025(L/h/d) 376.88 260.40 192.92 248.01 modif. por 260.40
Consumo prom.2025(Lt./seg.) 120.31 53.53 37.19 211.03 18"
Pob.Total año 2030(Hab. ) 28,267 17,768 29,044 75,079 17,768
Dotación Correg. 2030(L/h/d) 378.04 273.42 206.80 248.03 modif. por 273.42
Consumo prom.2030(Lt./seg.) 123.68 56.23 69.52 249.43 20"
Pob.Total año 2035(Hab. ) 28,826 17,776 54,119 100,721 17,776
Dotación Correg. 2035(L/h/d) 378.97 287.09 223.53 248.04 modif. por 287.09
Consumo prom.2035(Lt./seg.) 126.44 59.07 140.01 325.52 24"
Pob.Total año 2040(Hab. ) 29,300 17,784 54,119 101,203 17,784
Dotación Correg. 2040(L/h/d) 379.74 301.44 223.53 248.06 modif. por 301.44
Consumo prom.2040(Lt./seg.) 128.78 62.05 140.01 330.84 24"
Pob.Total año 2045(Hab. ) 29,712 17,792 114,512 162,016 17,792
Dotación Correg. 2045(L/h/d) 380.40 316.51 245.49 248.07 modif. por 316.51
Consumo prom.2045(Lt./seg.) 130.82 65.18 325.37 521.37 30"
% de Incremento (2045), con respecto al año de evaluación catastral (2019): 283.4%
Rpta. B) CAUDAL DE DEMANDA REAJUSTADA LA DOTACIÓN: 521.40 Lt./seg.
CUADRO 02.- POBLACIÓN Y CONSUMOS PROMEDIOS
Porcentaje que no es significativo en
el dimensionameinto de los
componentes, hecho que es
demostrable con la asignación de
diámetros económicos para cada
Caudal (Arocha: Relación Diámetros
vrs. Velocidad Económica); por lo que
no se aplicara la metodologìa
consistente en la construcciòn del
sistema por etapas(baterias),
recomendada para casos de
crecimiento explosivo durante el
periodo de diseño que generan gastos
iniciales altos al usar parámetros de
diseño correspondientes al último año,
que hacen enviable el proyecto por la
rentabilidad inicial.
Elaboramos el cuadro de Población y consumos Prom.usando la Expresión de CAPEN (G=k P 0.125 ), ACONDICIONADA A LOS PARÁMETROS DEL CRECIMIENTO POBLACIONAL DEL
PROYECTO, para calcular la demanda corregida para el año 2045, considerando el inicio de operación 2025
Se determina diámetros INICIALES DE DISEÑO (usando el cuadro de relación: Diámetro-Velocidad Económica, propuesto por Simón Arocha Ravelo-Normas INOS-Venezuela -1978 ó Esp. Técnicas del
fabricante AMANCO) a efectos de CONFIRMAR LA PROPUESTA DE EJECUCIÓN POR ETAPAS
Se observa que el Qprom. Total para el
año 2035 representa 2.834 del año de
base observación (2019)
-1,000,000
POBLACIÓN INC. Pob/year Polinómica (POBLACIÓN)
Rpta. C) ETAPAS CONSTRUCTIVAS A CONSIDERAR (Verif.2final): 1 ETAPA
Valores de k de la Expresión de CAPEN corregida de acuerdo a los datos poblacionales 2007 (*)
G = k P
0.125
Donde : G = Consumo por habitante en Lit/hab./día
P = Población en miles
k= 204.40 ( Ref.:Para una zona especifica de Venezuela - D.Lauria)
Zona "A:GH" 248.96 Zona "B:HI" 173.1 Zona "C:IJ" 135.73
POBLACIÓN EN ETAPA DE SATURACIÓN
Zona "A:GH"
DIÁMETRO DE DISEÑO AL INICIO DE OPERAC. DIÁMETRO FINAL DE DISEÑO
2025:
18"
2030-2035:
24"
Datos para Graficos : Zona "A:GH"
Año
2019 26,500 100.0% 375.00 100.0% 115.02 100.0%
2025 27,581 104.1% 376.88 100.5% 120.31 104.6%
2030 28,267 106.7% 378.04 100.8% 123.68 107.5%
2035 28,826 108.8% 378.97 101.1% 126.44 109.9%
2040 29,300 110.6% 379.74 101.3% 128.78 112.0%
2045 29,712 112.1% 380.40 101.4% 130.82 113.7%
Datos para Graficos : Zona "B:HI"
Año
2019 17,750 100.0% 248.00 100.0% 50.95 100.0%
2025 17,760 100.1% 260.40 105.0% 53.53 105.1%
2030 17,768 100.1% 273.42 110.3% 56.23 110.4%
2035 17,776 100.1% 287.09 115.8% 59.07 115.9%
Diámetro de conducción en función a la demanda sin considerar características físicas del
terreno: Captación-Línea de conducción-redes
% Consum.
Población
(Hab) % Poblac.
Dotación
(L./h/d)
Población
(Hab) % Poblac.
Dotación
(L./h/d) % Dotación
Consumo
(L/s.)
Por tener un incremento de habitantes por año con característica cercana al tipo intermedio(lineal) y tardío (logarítmica)
% Dotación
Consumo
(L/s.) % Consum.
90.0%
95.0%
100.0%
105.0%
110.0%
115.0%
2019 2025 2030 2035 2040 2045
Título del gráfico
% Poblac. % Dotación % Consum.
2040 17,784 100.2% 301.44 121.5% 62.05 121.8%
2045 17,792 100.2% 316.51 127.6% 65.18 127.9%
Datos para Graficos : Zona "C:IJ"
Año
2019 8,750 100.0% 178.00 100.0% 18.03 100.0%
2025 16,656 190.4% 192.92 108.4% 37.19 206.3%
2030 29,044 331.9% 206.80 116.2% 69.52 385.6%
2035 54,119 618.5% 223.53 125.6% 140.01 776.5%
2040 114,512 1308.7% 223.53 125.6% 140.01 776.5%
2045 318,784 3643.2% 245.49 137.9% 325.37 1804.6%
iii)
Se tiene el siguiente perfil de una línea de aducción por gravedad
Cuadro 01:
Puntos A B C D E F
Población
(Hab) % Poblac.
Dimensionamos la línea de conducción calculando el GASTO DE DISEÑO y la CARGA DISPONIBLE
Dotación
(L./h/d) % Dotación
Consumo
(L/s.) % Consum.
0.0%
50.0%
100.0%
150.0%
2019 2025 2030 2035 2040 2045
Título del gráfico
% Poblac. % Dotación % Consum.
0.0%
50.0%
100.0%
150.0%
2019 2025 2030 2035 2040 2045
Título del gráfico
% Poblac. % Dotación % Consum.
Cotas Mín. 1,912.00 1,788.00 1,855.00 1,600.00 1,900.00 1,550.00
D.Horiz.(Acum.) 0.00 300.00 750.00 1,250.00 2,000.00 2,500.00
Qpdb2019 = 184.00 l/s (Caudal promedio de demanda catastral base año de observación)
ANÁLISIS DE POSIBLES ETAPAS CONSTRUCTIVAS:
Año/Coefic. K1 K2 Qpd K1 Qpd Inc.K1Qpd K2 Qpd Inc. K2 Qpd K3Qpd Distor.CC+RV
2019 1.100 2.800 184.000 202.400 0.00000 515.200 0.00000 Usamos: 0.0 0.0 l/s
2020 1.100 2.800 188.510 207.361 4.96100 527.828 12.62800 Usamos: 0.0 0.0 l/s
2021 1.100 2.800 193.020 212.322 4.96100 540.456 12.62800 Usamos: 0.0 0.0 l/s
2022 1.100 2.800 197.530 217.283 4.96100 553.084 12.62800 Usamos: 0.0 0.0 l/s
2023 1.100 2.800 202.040 222.244 4.96100 565.712 12.62800 Usamos: 0.0 0.0 l/s
2024 1.100 2.800 206.550 227.205 4.96100 578.340 12.62800 Usamos: 0.0 0.0 l/s
2025 1.155 2.660 211.030 243.740 16.53465 561.340 -17.00020 Usamos: 0.0 0.0 l/s
2026 1.155 2.660 218.710 252.610 8.87040 581.769 20.42880 Usamos: 0.0 0.0 l/s
2027 1.155 2.660 226.390 261.480 8.87040 602.197 20.42880 Usamos: 0.0 0.0 l/s
2028 1.155 2.660 234.070 270.351 8.87040 622.626 20.42880 Usamos: 0.0 0.0 l/s
2029 1.155 2.660 241.750 279.221 8.87040 643.055 20.42880 Usamos: 0.0 0.0 l/s
2030 1.213 2.527 249.430 302.496 23.27498 630.310 -12.74539 Usamos: 0.0 0.0 l/s
2031 1.213 2.527 264.650 320.954 18.45805 668.771 38.46094 Usamos: 0.0 0.0 l/s
2032 1.213 2.527 279.870 339.412 18.45806 707.231 38.46094 Usamos: 0.0 0.0 l/s
2033 1.213 2.527 295.090 357.870 18.45805 745.692 38.46094 Usamos: 0.0 0.0 l/s
2034 1.213 2.527 310.310 376.328 18.45806 784.153 38.46094 Usamos: 0.0 0.0 l/s
2035 1.273 2.401 325.520 414.513 38.18465 781.460 -2.69378 Usamos: 0.0 0.0 l/s
2036 1.273 2.401 326.580 415.863 1.34979 784.004 2.54469 Usamos: 0.0 0.0 l/s
2037 1.273 2.401 327.640 417.213 1.34979 786.549 2.54469 Usamos: 0.0 0.0 l/s
2038 1.273 2.401 328.700 418.562 1.34979 789.094 2.54469 Usamos: 0.0 0.0 l/s
2039 1.273 2.401 329.760 419.912 1.34979 791.638 2.54469 Usamos: 0.0 0.0 l/s
2040 1.337 2.281 330.840 442.352 22.43963 754.519 -37.11885 Usamos: 0.0 0.0 l/s
2041 1.337 2.281 368.950 493.307 50.95524 841.434 86.91433 Usamos: 0.0 0.0 l/s
2042 1.337 2.281 407.060 544.262 50.95524 928.348 86.91433 Usamos: 0.0 0.0 l/s
2043 1.337 2.281 445.170 595.218 50.95524 1015.262 86.91433 Usamos: 0.0 0.0 l/s
2044 1.337 2.281 483.280 646.173 50.95524 1102.177 86.91433 Usamos: 0.0 0.0 l/s
2045 1.404 2.167 521.370 731.956 85.78356 1129.593 27.41644 Usamos: 732.0 1,129.6 l/s
Los valores establecidos en los estudios de validación y calibración de los parámetros de diseño de los coeficientes diarios y horarios son:
Se solicita predimensionar la línea de conducción con la combinación de diámetros de mayor eficiencia según las características de los parámetros de demanda, tipo de material de las
800.000
1000.000
1200.000
Caudales de Diseño (Qp1, Qp2)
0.00000
20.00000
40.00000
60.00000
80.00000
100.00000
Inc.K1Qpd
Q1diseño = 732.0 l/s
CARGAS DISPONIBLES:
Todo el tramo hasta donde empiezan las viviendas rurales AH : 1912 - 1550 = 362.00 m.
Punto crítico E, tramo AE : 1912 - 1900 12.00 m.
Cota Máx. 1,912.00 (Con fines Académicos)
Puntos A B C D E F
Cotas Mín. 1,912.00 1,788.00 1,855.00 1,600.00 1,900.00 1,550.00
D.H.(Parcial) 0.00 300.00 450.00 500.00 750.00 500.00
D.H.(Acum.) 0.00 300.00 750.00 1,250.00 2,000.00 2,500.00
D.I.(Parcial) 0.00 324.62 454.96 561.27 807.77 610.33
D.I.(Acumul.) 0.00 324.62 779.58 1,340.85 2,148.62 2,758.95
DESNIVELES: 124.00 57.00 312.00 12.00 362.00
Fin. PD
Inicio de
Operación
Qdis.(2045)= 732 l/s (PD=20) 2045 2025
Cota nivel max. = 1912.00 m
Cota nivel min. = 1912.00 m
A : Fuente de abastecimiento
B : Primer punto de cambio de pendiente de la tubería
C : Segundo punto de cambio de pendiente de la tubería
D : Tercer punto de cambio de pendiente de la tubería
E : Cuarto punto de cambio de pendiente de la tubería (CRÍTICO)
F : Quinto BM final de línea de conducción (Planta de Tratamiento - Altura mínima de llegada : 5.00 m. VALIDAR
Pto. Crítico
A B C D E F
( Se considera que la Planta de tratamiento y el Reservorio se ubicaran en la zona adyacente al punto G y en cotas superiores al existir viviendas rurales a partir de este punto)
CÁLCULO DE TUBERÍAS
-200.000
0.000
200.000
400.000
600.000
2019
2020
2021
2022
2023
2024
2025
2026
2027
2028
2029
2030
2031
2032
2033
2034
2035
2036
2037
2038
2039
2040
2041
2042
2043
2044
2045
K1 Qpd K2 Qpd Inc.K1Qpd Inc. K2 Qpd -50.00000
0.00000
50.00000
100.00000
Inc. K2 Qpd
D. HORIZ (Acum.) 0.00 300.00 750.00 1,250.00 2,000.00 2,500.00
D. INCLIN (Acum.) 0.00 324.62 779.58 1,340.85 2,148.62 2,758.95
COTA 1,912.00 1,788.00 1,855.00 1,600.00 1,900.00 1,550.00
12.00 362.00
DIÁMETRO DE TUBERÍA
Cota mínima de carga de agua sobre la tubería en la fuente de abastecimiento 1,912.00 m.
ΔH : Diferencia de elevación entre A y D
α : Coeficiente del diámetro
L : Longitud total
Q : Caudal de diseño
ΔH = 362.00 m Cota de nivel mínimo de fuente de abastecimiento - Cota de reservorio
LAF = 2,758.95 m
α = 6.59E-07 Coeficiente del diámetro calculado
POSIBLE COMBINACIÓN DE DIAMETROS
α : Coeficiente del diámetro
C : Coeficiente de rugosidad del material de la tubería
D : Diametro de la tubería
Buscando un coeficiente menor y un coeficiente mayor y próximos al coeficiente calculado
C = 150
DIAMETRO
(Pulg.)
α
0.50 4.7400E+00
0.75 6.5798E-01
1.00 1.6209E-01
1.25 5.4677E-02
2.00 5.5430E-03
2.50 1.8698E-03
3.00 7.6945E-04
4.00 1.8955E-04
6.00 2.6313E-05
8.00 6.4820E-06 α 2 : Coef. del diametro menor
10.00 2.1866E-06 (Final)
12.00 8.9980E-07 = α 2 (comercial)
13.1310 5.8030E-07 = α calc. (teórico) C= 150
14.00 4.2473E-07 = α 1 (comercial)
16.00 2.2166E-07
20.00 7.4773E-08
24.00 3.0770E-08 = α 2 (comercial)
24.4641 2.8030E-08 = α calc. (teórico) C= 150
Para tubería GRPR(Poliester reforzado con fibra de vidrio rectificado) C =140
85
.
1
LQ
H



87
.
4
85
.
1
3
10
72
.
1
D
C 



36.00 4.2714E-09 = α 1 (comercial)
48.00 1.0522E-09 α 1 : Coef. del diametro mayor
60.00 3.5495E-10 (Final)
75.00 1.1973E-10
90.00 4.9272E-11
120.00 1.2138E-11
150.00 4.0944E-12
Los posibles diámetros a combinar en el tramo AF es de 14" y 12" de forma preliminar
Luego de la verificación de conducción en elpunto crítico E, se tiene, que:
Los diámetros a combinar son de 36" y 24" de forma preliminar
PREDIMENSIONAMIENTO Y ANÁLISIS DEL TRAMO TOTAL AF:
VERIFICACIÓN DE LA APLICACIÓN DE LA FÓRMULA DE HAZEN Y WILLIAM:
Se debe cumplir: Tipo de flujo : Turbulento Re > 2300 > 106
Velocidad < 3 m/seg Diámetros mayores a 2" 104
- 106
Tuberías GRP: Poliester reforzado con fibra de vidrio (GRP) PAVCO ; C=150
> 2 x 105
ACERO: C=120
Si el flujo es de tipo Laminar entonces usar ecuación de DARCY
Hallando la velocidad media: PVC : Alegret Breña & Martinez Valdés, 2019: 0.05 m (2" )< D < 1.85 m (74")
3.05 m/s > V
V : Velocidad media del flujo 5° < T° < 25°
Q : Caudal de diseño
A : Area de la sección tubería
A = 0.65669289 m2
36.00 pulgadas Antes Re>2,300 A = 0.29186351 m2
24.00 pulg. Antes Re>2,300
V = 1.11 m/seg OK usar H y W si 10,0000<Re<1,000,000 V = 2.51 m/seg OK usar H y W si 10,0000<Re<1,000,000
Tipo de flujo :
Re : Velocidad media del flujo
V : Caudal de diseño
D : Area de la sección tubería
υ : viscocidad del flujo
36.00 pulgadas
Re = 112,776 OK usar H y W
Es correcto usar Hazen William
Tuberías CCP: Cilindro de acero revestido con concreto o mortero de cemento, reforzados helicoidalemnte con
varilla de acero al carbón . C=120

D
V 

Re
A
Q
V 
DETERMINAMOS LAS LONGITUDES ÓPTIMAS COMBINANDO DIÁMETROS
ΔH = 362.00 m Cota de nivel mínimo de carga de agua en fuente de abastecimiento - Cota de reservorio
Qdis.(2045)= 732 Lts/seg (PD=20)
LAF= 2,758.95 m
α 1 : 4.2473E-07 14.0 pulg. Coef. del diametro mayor (Inicial)
α 2 : 7.6945E-04 12.0 pulg. Coef. del diametro menor (final)
L'AF= 1.05 LAF= 2896.8975 m
J = 245.13 m OK 116.87
X : Distancia correspondiente al diametro mayor
ΔH : Diferencia de elevación entre A y E
L' : Longitud afectada en un 5%
ΔH = 362.00 m Cota de nivel mínimo de carga de agua en fuente de abastecimiento - Cota de reservorio
ΔHpt= 1.00 m Presión mínima en cualquier punto de la tubería por conducción
X = 1662 m Long. De tubería mayor : 14.00 Pulg. :α 1 4.2473E-07
L-X = 1,097.00 m Long. De tubería menor : 12.00 Pulg. :α 2 8.9980E-07
2,759.00
OK, ES PROCEDENTE LA COMBINACIÓN DE DIÁMETROS
CHEQUEANDO PARA LA LONGITUD CRÍTICA AE
ΔH : Diferencia de elevación entre A y D
α : Coeficiente del diámetro
L : Longitud total
Q : Caudal de diseño
CARGA DISP.
ΔHAE = 12.00 m Cota de nivel mínimo de carga de agua en fuente de abastecimiento - Cota en E
LAE= 2,148.62 m
α = 2.8033E-08 Coeficiente del diámetro calculado 2.8033E-08
Requisito : J < ΔH
CONSIDERANDO : 36.00 Pulg. :α 1 4.2714E-09
J = 1.83
12.00 Requisito : J < ΔH OK
10.17 COMBINAR DIÁMETROS PARA OPTIMIZAR EL DISEÑO
L'AE= 1.05 LAE= 2256.05 m
ΔH = 12.00 m Cota de nivel mínimo de carga de agua en fuente de abastecimiento - Cota en E RESP. 1: TRAMO INIC. 36.00 pulg.
ΔHpt= 1.00 m Presión mínima en cualquier punto de la tubería por conducción RESP. 2: TR.AE DE 24" = 346.00 m
CARGA DISPONIBLE ΔHAE =
EVALUAMOS LA DIFERENCIA
ΔHAE - J
 
2
1
85
.
1
85
.
1
2 '









Q
Q
L
H
X
85
.
1
1 Q
L
J AE


85
.
1
LQ
H



85
.
1
1 Q
L
J AF


X = 346 m Long. De tubería mayor : 36.00 pulg. :α 1 4.2714E-09 Rpta. D) 36.00 pulg.
L-X = 1,802.60 m Long. De tubería menor : 24.00 pulg. :α 2 3.0770E-08
2,148.60 Para las redes 14.00 pulg. :α 2 4.2473E-07 RESP. 3: TRAMO FINAL 24.00 pulg.
OK, ES PROCEDENTE LA COMBINACIÓN DE DIÁMETROS RESP. 4: TR.AE DE 24" = 1,802.60 m
CHEQUEO TRAMO CRÍTICO AE CON LA COMBINACIÓN DE DIÁMETROS: TOTAL TRAMO: AE 2,148.60 m.
TOTAL TRAMO: EF 610.33 m.
TOTAL TRAMO: AF 2,758.93 m.
J = 11.34
12.00 OK
0.66 AUMENTAR LONGITUD DE DIÁMETRO MAYOR PARA OPTIMIZAR EL DISEÑO
ALTURA MÍNIMA DE CARGA EN CADA NUDO 0.5 mca
CHEQUEANDO POR TRAMOS:
Cota mínima de carga de agua sobre la tubería en la fuente de abastecimiento 1,912.00 m.c.a.: metros de columna de agua
TRAMO AB
Cota Tub. B= 1,788.00
ΔHAB = 124.00 Cota de nivel mínimo de carga de agua en fuente de abastecimiento - Cota en B
LAB = 324.62 36.00 Pulg.
L'AB=1.05* LAB = 340.85
J = 0.29 OK Requisito : J < ΔH
Cota Piez B= 1911.71
Cota Tub. B= 1788.00
m.c.a.= 123.71
Altura mín. 0.50 OK
TRAMO AC
Cota Tub. C= 1,855.00
ΔHAC = 57.00 Cota de nivel mínimo de carga de agua en fuente de abastecimiento - Cota en C
LAC = 779.58 36.00 Pulg.
L'AC=1.05* LAC = 818.56
J = 0.70 OK Requisito : J < ΔH
Cota Piez C= 1911.30
Cota Tub. C= 1855.00
m.c.a.= 56.30
Altura mín. 0.50 OK
TRAMO AD
Cota Tub. D= 1,600.00
ΔHAD = 312.00 Cota de nivel mínimo de carga de agua en fuente de abastecimiento - Cota en D
LAD = 1,340.85 36.00 Pulg.
CARGA DISPONIBLE ΔHAE =
EVALUAMOS LA DIFERENCIA
ΔHAE - J
85
.
1
1 Q
L
J AC


85
.
1
1 Q
L
J AB


 
  85
.
1
2
1 Q
L
L
L
J X
AE
X 

 

85
.
1
1 Q
L
J AD


L'AD=1.05* LAD = 1,407.89
J = 1.20 OK Requisito : J < ΔH
Cota Piez D= 1910.80
Cota Tub. D= 1600.00
m.c.a.= 310.80
Altura mín. 0.50 OK
TRAMO AE
Cota Tub. E= 1,900.00
ΔHAE = 12.00 Cota de nivel mínimo de carga de agua en fuente de abastecimiento - Cota en D
LAE = 2,148.62 36.00 Pulg. + 24.00 Pulg. COMBINACIÓN DE DIÁMETROS
L'AE=1.05* LAE = 2,256.03 36.00 Pulg. 346.00 m
J = 11.34 OK Requisito : J < ΔH 24.00 Pulg. 1802.60 m
Cota Piez E= 1900.66 2148.60 m
Cota Tub. E= 1900.00
m.c.a.= 0.66
Altura mín. 0.50 OK
TRAMO AF
Cota Tub. F= 1,550.00
ΔHAF = 362.00 Cota de nivel mínimo de carga de agua en fuente de abastecimiento - Cota en F
LAF = 2,758.95 36.00 Pulg. + 24.00 Pulg. COMBINACIÓN DE DIÁMETROS
L'AF=1.05* LAF = 2,896.90 36.00 Pulg. 346.00 m
J = 15.09 OK Requisito : J < ΔH 24.00 Pulg. 2412.95 m
Cota Piez F= 1896.91 2758.95 m
Cota Tub. F= 1550.00
m.c.a.= 346.91
Altura mín. 0.50 OK
CONSIDERANDO UN SOLO DIAMETRO PARA TODA LA LONGITUD (AF)
Cota Tub. F= 1,550.00
ΔHAF = 362.00
LAF = 2,758.95
L'AF=1.05* LAF = 2,896.90 36.00 Pulg.
J = 2.47
Cota Piez F= 1909.53
Cota Tub. F= 1550.00
Queda demostrado que un 5% de aumento de longitud equivale a 1 metro
85
.
1
1 Q
L
J AE


 
  85
.
1
2
1 Q
L
L
L
J X
AE
X 

 

 
  85
.
1
2
1 Q
L
L
L
J X
AE
X 

 

m.c.a.= 359.53 - 346.91 = 12.62 mca AHORRO
Altura mín. 0.50 OK m.c.a.: metros de columna de agua
Qd = 732.00 l/s L(acum.) = 2,758.95 m
USAR TUBERÍA
36 pulgadas 36" 346.00 m. POLIETILENO DE ALTA DENSIDAD (HDPE)
POLIVINILO DE CLORURO (PVC) PESADO
RESP. 1: TRAMO FINAL 24 pulgadas 24" 2,413.00 m. POLIVINILO DE CLORURO (PVC) PESADO
TOTAL LONGITUD AE
AF 2,759.00 m
CLASE DE TUBERÍA
Cota Inicio(máx.) 362.00 m.
USAR TUBERIA PVC CLASE 5, 7.5, 10, 15 DE 8¨ y 6 ¨ s/esquema
SE ANALIZA EL TRAMO HASTA EL PUNTO CRÍTICO AE:
En función de la carga disponible entre A y E α AE =
En función a los diámetros para tubería PVC α X" = 0
C = 150
α Y" = 0
α Z" = 0
En función a los diámetros para tubería ASBESTO α N" = 0
CEMENTO
C = 120 α M" = 0
α R"= 0
En función a los diámetros para F tubería F G α S" = 0
CARGA DISPONIBLE EN
PARA DETERMINAR LA CLASE DE TUBERÌA A USAR SE INICIARA CON CLASE C-5, CONSIDERAN LA COTA MAXIMA DEL NIVEL DE AGUA, COMO EL PUNTO D TIENE DIFERENCIA DE COTA CRÍTICA
(DIFERENCIA DE COTAS MAYOR CON LA CARGA HIDRÁULICA), DE SPBREPASA VALORES DE 250 M (C-25) SE CONSIDERARA LA CONSTRUCCIÓN DE UN PUENTE USANDO TUBERÍA CLASE 25
C = 110
α T" = 0
α R"= 0
J AE = 15.09 ( Usando ΦHDPE 8" Y 6" ) m.c.a F = 346.91 Requerim. M.c.a F(Pta.Tr) = 5.00 OK VALIDAR
J EF = XXXXXX ( Usando Φfºgº AE1' ,Φ pvcE1E2" y Φ pvc E2F" )
Si ΔEEF = 350.00 m SE NECESITA CÁMARAS O VÁLVULAS ROMPEPRESIÓN
iv)
POR DATO SABEMOS QUE EN EL TRAMO ABCDE SE USARA TUBERÍA DE HDPE, POR LO QUE CHEQUEAMOS LA PERDIDA DE CARGA JAC. CON LOS α CORRESPONDIENTES A LA
COMBINACIÓN DE DIÁMETROS DE A hacia F USANDO SÓL TUBERÍA HDPE-PVC-ITINTEC-DIÁMETRO INTERNO EN PULGADAS.
NECESITAMOS PRESIÓN SOLO PARA PASAR EL PUNTO E LUEGO DE ENCONTAR UNA DIFERENCIA MAYOR A 50 METROS DE ALTURA DE
(Topografía Plana de
Poblaciones A, B y C, sólo
con fines académicos)
Con éstos datos elaboramos el perfil longitudinal de la línea de Conducción
240
250
260
270
280
290
300
310
320
330
340
350
Cotas Mín.
Cotas
PLANTA DE
TRATAMIENTO
PLANO ESTÁTICO DE PRESIONES
360 m = PLANO ESTÁTICO DE PRESIÓN LIMITANTE (100 Lb./Pulg2 <>70 m. de agua)
PLANO ESTÁTICO LIMITE DE PRESIONES PARA DETERMINAR LA CLASE DE TUBERÍA: 50 M. DE ALTURA DE AGUA DESDE EL PUNTO A.
PLANO ESTÁTICO LIMITE DE PRESIONES PARA DETERMINAR LA CLASE DE TUBERÍA: 75 M. DE ALTURA DE AGUA DESDE EL PUNTO A.
Pob. A Pob. B Pob. C
Cota Máx. 1,912.00
Puntos A B C D E F G H I J
Cotas Mín. 1,912.00 1,788.00 1,855.00 1,600.00 1,900.00 1,550.00 1,550.00 1,550.00 1,550.00 1,550.00
D.H.(Parcial) 0.00 300.00 450.00 500.00 750.00 500.00 1,000.00 1,000.00 1,000.00 1,000.00
D.H.(Acum.) 0.00 300.00 750.00 1,250.00 2,000.00 2,500.00 3,500.00 4,500.00 5,500.00 6,500.00
D.I.(Parcial) 0.00 324.62 454.96 561.27 807.77 610.33 1,000.00 1,000.00 1,000.00 1,000.00 (Verific.Datos)
D.I.(Acumul.) 0.00 324.62 779.58 1,340.85 2,148.62 2,758.95 3,758.95 4,758.95 5,758.95 6,758.95
Φ 36" 36"- 24" 24" 24" 24" 24" 14"
MATERIAL Φ PVC PVC PVC
J 0.00 0.29 0.70 1.20 11.34 15.09
Cota Piezom. 1,912.00 1,911.71 1,911.30 1,910.80 1,900.66 1,896.91 1,896.91 1,896.91 1,896.91 1,896.91
VERIFICACIÓN 0.00 123.71 56.30 310.80 0.66 346.91 346.91 346.91 346.91 346.91
OK OK OK OK OK
CLASE Φ 5 5,7.5,10,15,20,25 5,7.5,10,15,20,25 5,7.5,10,15,20,25 5,7.5,10,15,20,25 5,7.5,10,15,20,25 5,7.5 5,7.5 5,7.5 5,7.5
D.V.(parcial) 124.00 0.00 67.00 -188.00 112.00 -350.00 -350.00 -350.00 -350.00
COTA PIEZOMÉTRICA F: 1,896.91 m
COORDENADAS DE UBICACIÓN DE EMPALMES DE DIÁMETROS DE TUBERÍAS, OBRAS DE ARTE, PUENTES Y CAMARAS O VÁLVULAS ROMPE-PRESIÓN
1ER. PRE-DIMENSIONAMIENTO CON CARGAS DINÁMICAS EN EXTREMOS (CRÍTICAS) OBRAS DE ARTE COTA
Intersecciones de Clases de Tuberías C= 150 PUENTE 1
Altura de agua (mca) para C-5, Tramo CD: 50 mca CAMARA ROMPE PRESION 1
Cota inicial para usar C-5 +1912 1,912.00 m CAMARA ROMPE PRESION 2
Cota final para usar C-5 +1912-1.2-50= 1,860.80 m CAMARA ROMPE PRESION 3
Por semejanza de triángulos CAMARA ROMPE PRESION 4
HDPE (Inst. Exterior)
150
160
170
180
190
200
210
220
230
240
A B C D E F G H I J
Cotas
Mín.
PLANTA DE
TRATAMIENTO
Y RESERVORIO
PUENTE (NO)
CAJAS ROMPEPRESIÓN (NO)
CISTERNA, CASETA
DE BOMBEO Y
RESERVORIO
PLANO ESTÁTICO LIMITE DE PRESIONES PARA DETERMINAR LA CLASE DE TUBERÍA: 100 M. DE ALTURA DE AGUA DESDE EL PUNTO A.
PLANO ESTÁTICO LIMITE DE PRESIONES PARA DETERMINAR LA CLASE DE TUBERÍA: 150 M. DE ALTURA DE AGUA DESDE EL PUNTO A.
CAMARA ROMPE PRESION 5
Puntos C D
DH.acum. 750.00 1,250.00 CAMARA ROMPE PRESION 6
Cotas 1,855.00 1,600.00 CAMARA ROMPE PRESION 7
DIacum. 779.58 1,340.85
x'CD CAMARA ROMPE PRESION 8
X'CD -11.37 m CAMARA ROMPE PRESION 9
C1 792.34 DIparc.C-C1= 12.8 m CAMARA ROMPE PRESION 10
1,860.80 DIacum.A-C1= 792.34 m OK CAMARA ROMPE PRESION 11
CAMARA ROMPE PRESION 12
Distancia inclinado acum. A-C1 Clase C5= 816.00 m.
Tubería C5 de diámetro: 36" 346.00 m.
Tubería C5 de diámetro: 24" 470.00 m.
Altura de agua (mca) para C7.5, Tramo CD: 75 mca CAMARA ROMPE PRESION 13
Cota inicial para usar C-7.5 +1912 1,912.00 m CAMARA ROMPE PRESION 14
Cota final para usar C-7.5 +1912-1.2-75= 1,835.80 m CAMARA ROMPE PRESION 15
Por semejanza de triángulos CAMARA ROMPE PRESION 16
CAMARA ROMPE PRESION 17
Puntos C D CAMARA ROMPE PRESION 18
DH.acum. 750.00 1,250.00 CAMARA ROMPE PRESION 19
Cotas 1,855.00 1,600.00 CAMARA ROMPE PRESION 20
DIacum. 779.58 1,340.85 CAMARA ROMPE PRESION 21
x''CD RESERVORIO
X''CD 37.65 m
C2 821.84 DIparc.C-C2= 42.3 m REQUERIMIENTO FINAL:
1,835.80 DIacum.A-C2= 821.84 m OK Norma de Fabricación ASTM ó ITINTEC
Tubería C5-36" incluído 3% desperd.= Unid. 163.2 tubos
Distancia inclinado acum.A-C2 C5-C7.5= 821.84 m.
Tubería C5 de diámetro: 36" 346.00 m.
Tubería C5 de diámetro: 24" 475.84 m.
Tubería C5 de diámetro: 24" 470.00 m.
Tubería C7.5 de diámetro: 24" 5.84 m.
Altura de agua (mca) para C10, Tramo CD: 100 mca
Cota inicial para usar C-7.5 +1912 1,912.00 m
Cota final para usar C-7.5 +1912-1.2-100= 1,810.80 m
Por semejanza de triángulos
Puntos C D
DH.acum. 750.00 1,250.00
Cotas 1,855.00 1,600.00
DIacum. 779.58 1,340.85
x'''CD
X'''CD 86.67 m
C3 876.87 DIparc.C-C3= 97.3 m
1,810.80 DIacum.A-C3= 876.87 m OK
DHparc.x-D= 413.33 m
DIacum.C3-D= 463.98 m Para verificación final
Distancia inclinado acum.A-C3 Clase C10= 876.87 m.
Tubería C5 de diámetro 36" 346.00 m.
Tubería C5 de diámetro 24" 530.87 m.
Tubería C5 de diámetro: 24" 470.00 m.
Tubería C7.5 de diámetro: 24" 5.84 m.
Tubería C10 de diámetro: 24" 55.03 m.
Rpta. E) 1810.80 msnm
Altura de agua (mca) para C10, Tramo DE: 100 mca Ubicación del Puente: P1
Cota inicial para usar C-7.5 +1912 1,912.00 m Punto. C3-Cota inicial C10-4 ": 1,810.80
Cota final para usar C-7.5 +1912-1.2-100= 1,810.80 m Punto. D3-Cota final C10-4 ": 1,810.80 (Con fines académicos)
Por semejanza de triángulos Longitud efectiva tramo C3-D3: 940.33 m
Puntos D E
DH.acum. 1,250.00 2,000.00
Cotas 1,600.00 1,900.00
DIacum. 1,340.85 2,148.62
x'DE
X'DE 223.00 m
306.86 D3 DIparc.E-D3= 306.86 m
1,810.80 DIacum.E-D3= 306.86 m
DHparc.x-D3= 527.00 m
DIacum.A-D3= 1817.20 m OK
DIacum.D-D3= 500.91 m Para verificación final
VERIFICACIÓN DE DIÁMETROS Y CLASE DE TUBERÍAS HASTA EL PUENTE P1: (BM1)
Distancia inclinada acum. A-D3: 1817.20 m
Tubería C5 de diámetro 36" 346.00 m. Clase C5
Tubería C5 de diámetro 24" 470.00 m. Clase C5
Tubería C7.5 de diámetro 24" 60.87 m. Clase C7.5
Tubería C10 de diámetro 24" 940.33 m. Clase C10 Puente 01
1,817.20 m. OK
Altura de agua (mca) para C7.5, Tramo DE: 75 mca
Cota inicial para usar C-7.5 +1912 +1912 m
Cota final para usar C-7.5 +1912-11.34-75= 1,825.66 m
Por semejanza de triángulos
Puntos D E
DH.acum. 1,250.00 2,000.00
Cotas 1,600.00 1,900.00
DIacum. 1,340.85 2,148.62
x''ED
X''DE 185.85 m
292.34 D2 C5: DIparc.E-D2= 292.34 m
1,825.66 DIacum.E-D2= 292.34 m
DHparc.x-D2= 564.15 m
DIacum.A-D2= 1854.35 m OK
Distancia inclinada acum. A-D2: 1854.35 m
Tubería C5 de diámetro 36" 346.00 m. Clase C5
Tubería C5 de diámetro 24" 470.00 m. Clase C5
Tubería C7.5 de diámetro 24" 60.87 m. Clase C7.5
Tubería C10 de diámetro 24" 940.33 m. Clase C10 Puente 01
Tubería C7.5 de diámetro 24" 37.15 m. Clase C7.5
Tubería C5 de diámetro 24" 292.34 m. Clase C5 DIparc.E-D2=
VERIFICACIÓN Y CONTROL SECUENCIAL (BM2):
Distancia inclinada acum. A-E: 2146.69 m +TR:C3-D3: 964.89 = 3111.58 - 940.33 = 2,171.3 m
COTA EN EL PTO. E. = 2,148.62 m
ERROR ACUMULADO POR USAR PRESIONES DINÁMICA EN LOS PTOS. EXTREMOS A CORREGIR SI GENERARN DISTORSIONES SIGNIFICATIVAS(TÉCNICO-ECONÓMICO) 22.63 m
COORDENADAS DE UBICACIÓN DE CÁMARAS O VÁLVULAS ROMPE-PRESIÓN Ubicación de la 1CR
Se usara la Formula de Manning
D = 1.548(nQ/s^1/2)^3/8
Cota dinámica de llegada al Pto. E. 1,900.66 m Diam.(pulg.) D(m), Q(m3/seg.), S(m/m)
Cota topográfica del pto. E 1,900.00 m 0 0.00 n(pvc) = 0.009
mca disponible en el pto.E. 0.66 m. ¡RECALCULAR¡ 0.00 0.00 #¡DIV/0! m.
0.00 0.00 #¡DIV/0! pulgadas
Cota dinámica de llegada al Pto. F. 1,896.91 m Se usara : 8"
Cota topográfica del pto. E 1,550.00 m COTA USANDO 8" 3,679.0 msnm.
mca disponible en el pto.E. 346.91 m. DI 1CR 78.6 m.
mca mín. requerido en E (Reserv. Transv.) 5.00 m OK DH 1CR 35.2 m.
mca mín. requerido en E (Reserv. Distrib.) 75.00 m OK
CONCLUSIONES INICIALES:
DATOS TOPOGRÁFICOS DE LAS ZONAS A (tramo FG), B(tramo GH) Y C(tramo HI):
Tramo longitudinal FG: 1,000.00 m Cota G : 1,550.00
Por la escesiva carga hidráulica generada en el tramo EF SE CONSIDERARAN LAS CONSTRUCCIÓN DE CAMARAS ROMPEPRESIÓN O ADQUISICIÓN DE VÁLVULAS REDUCTORAS
Se plantean DOS ESCENARIOS CONSTRUCTIVOS, 1er. Escenario) La construcción de un sólo rservorio principal en el Pto E para abastecer a las tres poblaciones A,B, y C, del tipo semi-
enterrado por la capacidas de carga hidráulica de llegada, reduciendo las presiónes con válvulas en la distribución de la red de agua para no sorepasar los 50 mca exigidos en el reglamento.
2do. Escenario), del Reservorio Principal en E se distribuirá a reservorios zonales en la población A(tramo GH), población B (tramo HI) y población C (tramo IJ), para redistribuir el agua por
gravedad.
Tramo longitudinal GH: 1,000.00 m Cota H : 1,550.00
Tramo longitudinal HI: 1,000.00 m Cota I : 1,550.00
Pto A: 1,912.00 24" ZONA "A" 14" ZONA "B" 14" ZONA "C" 14"
Puntos E Cota ResP F G H I
DH.acum. 2,000.00 2,500.00 3,500.00 4,500.00 5,500.00
Cotas 1,900.00 1868.94 1,550.00 1,550.00 1,550.00 1,550.00
DIacum. 2,148.62 1784.33 2,758.95 3,758.95 4,758.95 5,758.95
J 11.34 1699.71 15.09 99.71 184.33 268.94
Cota Piezom. 1,900.66 1,896.91 1812.29 1727.67 1643.06 Manteniendo el NAM=Cterr.E
Carga Hid. (mca) 0.66 346.91 262.29 177.67 93.06 OK Art. 4.5 OS.050-RNE
200.29 115.67 31.06
MÁXIMA PRESIÓN SEGÚN REGLAMENTO (mca) 62.00 62.00 62.00
NUEVA CARGA PIEZOMÉTRICA PARA USAR UN SOLO RESERVORIO 50.00 50.00 50.00
Rpta. F) 1868.90 msnm
E
C3 0.00
1,550.00
ZONA "A" ZONA "B" ZONA "C"
F G H I
SE PROCEDE A EFECTUAR LOS CÁLCULOS CONSIDERANDO UN RESERVORIO PRINCIPAL, PARA ABASTECER A LA POBLACIÓN, A, B Y C, TIPO ENTERRADO UBICADO EN EL PUNTO : E
ESCENARIO DE SISTEMA DE DISTRIBUCIÓN DE AGUA 1: DISTRIBUCIÓN POR GRAVEDAD DE 1 SOLO RESERVORIO PRINCIPAL
AÑO BASE DE OBSERVACIÓN ESTADISTISTICA-CATASTRAL 2019 Año inicio de Operac. 2025 Año final del Pdiseño 2045
i)
POBLACIÓN TOTAL 2045
100.0%
162,016 habitantes
35,307 Viv.
Viviendas A : Densidad poblacional promedio: 8 Hab. / Viv. 18.3% 29,712 N° Viviendas 3,714
Viviendas B : Densidad poblacional promedio: 6 Hab. / Viv. 11.0% 17,792 N° Viviendas 2,965
Viviendas C : Densidad poblacional promedio: 4 Hab. / Viv. 70.7% 114,512 N° Viviendas 28,628
162,016
CUADRO 01a.-
SE PUEDE CONSIDERAR UN SOLO
RESERVORIO PRINCIPAL
Se deberá verificar la dotación promedio CATASTRAL, en función al consumo promedio cuantificado por tipo de EDIFICACIÓN Y HABITANTE, así como el
consumo promedio en litros/seg, para el año final del periodo de diseño establecido.
PLANTA DE
TRATAMIENTO Y
RESERVORIO
PRINCIPAL
ENTERRADO
CISTERNA +
RESERVORIO
ZONAL A
CISTERNA +
RESERVORIO
ZONAL B
CISTERNA +
RESERVORIO
ZONAL C
85
.
1
LQ
J 

DEMANDA POBLACIONAL: Zona "A" Zona "B" Zona "C" Total
Densid.prom.pob.(hab./edif.)RMD3-R3,R4 8.00 6.00 4.00 (Dato)
Pob. total año 2045 s/modif.usos( hab.) 29,712 17,792 114,512 162,016
Dotac. 2045 s/Rgmto-OS100correg.(l /hab./día) 380.40 316.51 245.49 (Clima cálido)
Cons. Prom. 2045 /modif. ( l/s) 130.82 65.18 325.37 521.36 100.0%
CUADRO 01b.-
DEMANDA CATASTRAL: Zona "A" Zona "B" Zona "C" Total
Densidad prom.poblac.(hab./edif.)
8.00 6.00 4.00
Rptas:P4,P5,P6 Pob.RESID.Total de Saturación, Pd=?? ( hab.)
0 0 0 0
Rptas:P7,P8,P9 Pob.Tot.(RESID+FLOT.COM) Satur.Pd=??(hab.)
0 0 0 0
Dot.2019 s/Rgmto-os100(Lt./hab./día)
375.00 248.00 178.00 (Clima cálido)
Dot. año catast. 2045 del Pd=20 (l/hab./día)
380.40 316.51 245.49
Qp.pob.RESID. 2045 c/mod. Pd=???(l/s)
128.96 51.07 235.92 415.94
Qp.pob.RESID.+FLOT.2045c/mod.Pd=20(l/s)
0.00 0.00 0.00 0.00 Para zonas comerciales e indutriales
Qp.pob. año 2045 Final Pd=20 (l/s)
130.82 65.18 325.37 521.36
Qp catast.pob.sat.año 2045 Pd=20(l/s)
???
PÉRDIDAS EN LA DISTRIB. p/dato (Dependerá de gestión de optimiz.) 15.0% 599.56 600 l/s
ii)
Caudal ofertado por el sistema (l/s): 1,000.00 Caudal de demanda: Zona A+B+C(l/s): 600.00
1.66667 166.67% 0.60000 60.00%
Reservorio existente el 2019 (Dato) 100 m3
GASTO CUADRO 02.- LOCALIDAD ß ( Consumo promedio año 2045)
PROM. GASTO VOLUM. PRODUCC. VOLUM. DE PRODUCC. VOLUM. CONS.
HORAS @ 2 horas PROM @ 2HR. VOLUMEN ACUMUL. ACUM.24Hr.2017 CONS.RESERV. AC.REAL RESERV.REAL
% del Qprom. 2045 60.00% 60.00% 166.667% 166.667%
(Lt./seg.) (Lts./s.) (Lts.) (Lts.) (Lts.) (Lts.) (Lts.) (Lts.)
0 0 0
0 1.00% 6.000 54,000 54,000 2,160,000 2,106,000 3,600,000 3,546,000
1 1.50% 9.000 157,680 211,680 4,320,000 4,108,320 7,200,000 6,988,320
2 5.80% 34.800 816,480 1,028,160 6,480,000 5,451,840 10,800,000 9,771,840
3 11.00% 66.000 626,400 1,654,560 8,640,000 6,985,440 14,400,000 12,745,440
Norma OS.030:
Valmac. = Vregulac. + Vincendio + Vreserva
Vregulac. = Diagrama Masa ó 25% del Qprom. en 24 horas
Vreserva = 25% Vtotal
Vreserva = 33% (Vregulac. + Vincendio)
Vreserva = Qp x t ( 2 hr. < t > 4 hr. )
Vincendio = 50 m3 (para viviendas) ; GCIS con 3,000 m3 de
volumen aparente (Edif. de uso comercial o industrial)
Para determinar el caudal medio en Lt./seg. durante el año 2015, se procederá a desarrollar el cuadro que contiene las variaciones
Factor de Deficit y Superavit :
Efectuamos las estimaciones de las demanda por variaciones de consumo diarias en función a los datos suministrados del diario promedio horario y del diario
maximo horario en Lt./seg., a fin de determinar los valores de K2.
4 12.00% 72.000 648,000 2,302,560 10,800,000 8,497,440 18,000,000 15,697,440
5 75.00% 450.000 2,095,200 4,397,760 12,960,000 8,562,240 21,600,000 17,202,240
6 98.00% 588.000 2,872,800 7,270,560 15,120,000 7,849,440 25,200,000 17,929,440
7 175.00% 1,050.000 4,968,000 12,238,560 17,280,000 5,041,440 28,800,000 16,561,440
8 210.00% 1,260.000 5,248,800 17,487,360 19,440,000 1,952,640 32,400,000 14,912,640
9 195.00% 1,170.000 4,255,200 21,742,560 21,600,000 -142,560 36,000,000 14,257,440
10 155.00% 930.000 3,386,880 25,129,440 23,760,000 -1,369,440 39,600,000 14,470,560
11 170.00% 1,020.000 3,693,600 28,823,040 25,920,000 -2,903,040 43,200,000 14,376,960
12 320.00% 1,920.000 8,538,134 37,361,174 28,080,000 -9,281,174 46,800,000 9,438,826
13 155.00% 930.000 3,352,680 40,713,854 30,240,000 -10,473,854 50,400,000 9,686,146
14 80.00% 480.000 2,419,200 43,133,054 32,400,000 -10,733,054 54,000,000 10,866,946
15 32.00% 192.000 1,080,000 44,213,054 34,560,000 -9,653,054 57,600,000 13,386,946
16 18.00% 108.000 777,600 44,990,654 36,720,000 -8,270,654 61,200,000 16,209,346
17 18.00% 108.000 864,000 45,854,654 38,880,000 -6,974,654 64,800,000 18,945,346
18 22.00% 132.000 1,231,200 47,085,854 41,040,000 -6,045,854 68,400,000 21,314,146
19 35.00% 210.000 1,944,000 49,029,854 43,200,000 -5,829,854 72,000,000 22,970,146
20 55.00% 330.000 1,900,800 50,930,654 45,360,000 -5,570,654 75,600,000 24,669,346
21 33.00% 198.000 756,000 51,686,654 47,520,000 -4,166,654 79,200,000 27,513,346
22 2.00% 12.000 82,080 51,768,734 49,680,000 -2,088,734 82,800,000 31,031,266
23 1.80% 10.800 60,480 51,829,214 51,840,000 10,786 86,400,000 34,570,786
24 1.00% 6.000 51,829,214
POR DÍA 451.70 51,829,214 51,829.21 600.00 19,295,294 1,000.00 34,570,786
k1= 1.30 (Reglam.) 19,295 34,571 M3
(Lts./s.) (Lts.) (M3) Q act-zona A+B Vol Rs (Qp.2018: A+B) (*) Dato
600 k2prom.19: 3.20 0.00
0.60 m3
PRE-DIMENSIONAMIENTO DEL RESERVORIO PRINCIPAL SIN USAR LOS COEFICIENTES DE VARIACIÓN HORARIA
VRreserva= 25% Vralmacenam. Previa justificación. Se recomienda usar 25 % del Volumen total de almacenamiento
VRalmacenam.= VRconsumo + VRincendio + VReserva =
12,957.3036 + 50.000 + 25% Vralmacenam.
VRalmacenam.= (VRconsumo + VRincendio) / 0.75 = 17,343.071 m
3
17,343,071 litros Vreserva = 4,335.7679 m
3
VOLUMEN TOTAL RESERVORIO PRINCIPAL: 17,343.000 m
3
25.0%
Rpta. G) 17,343 m3
DIFERENCIA DEL VOLUMEN DEL RESERV. REQUERIDO s/QOFERTA. Y s/ QDEMANDA 15,275.49 79.2%
Para determinar los parámetros máximos de las variaciones horarias de consumo, se desarrollará el cuadro que contiene los datos en el día
14000000
CURVAS DE DEMANDA Y OFERTA DEL SISTEMA EN EL AÑO BASE DE OBSERVACIÓN: 2018
GASTO CUADRO 03.- LOCALIDAD ß(Consumo del día máximo año 2045)
PROM. GASTO VOLUM. PRODUCC. VOLUM. DE
HORAS @ 1 hora PROM @ 2HR. VOLUMEN ACUMUL. ACUM.24 Hr. CONSUM.RES.
% del Qprom. 100 % Dprom. 100% Dprom.
(Lt./seg.) (Lts./s.) (Lts.) (Lts.) (Lts.) (Lts.)
0 45.00% 0.00 0 0 0 0
2 55.00% 0.00 0 0 0 0
4 70.00% 0.00 0 0 0 0
6 75.00% 0.00 0 0 0 0
8 125.00% 0.00 0 0 0 0
10 130.00% 0.00 0 0 0 0
12 150.00% 0.00 0 0 0 0
14 140.00% 0.00 0 0 0 0
16 115.00% 0.00 0 0 0 0
18 120.00% 0.00 0 0 0 0
20 155.00% 0.00 0 0 0 0
22 120.00% 0.00 0 0 0 0
24 45.00% 0.00 0 0 0
POR DÍA 0.00 0 0.00 Reserv(187.05 l/s)= 0
0
2000000
4000000
6000000
8000000
10000000
12000000
0 2 4 6 8 10 12 14 16 18 20 22
PRODUCC. AC.REAL 1.192974705 (Lts.) VOLUM. ACUMUL.
0.838240741 PRODUCC. AC.REAL 1.192974705 (Lts.)
VOLUM. ACUMUL. 2035 (Lts.) PRODUCC. ACUM.24Hr.2035 0.838240741 (Lts.)
(Lts./s.) (Lts.) (M3) 0 m3 0.00%
0.00 k2dis.(2018): 0.00 (Dìa de Consumo Máximo)
k1= 1.30 (Reglam.) k2dis.2019: 3.20 (Dìa de Consumo Promedio)
2 Ptos.
El año: 2025
El año: 2045
Único sistema de suministro: BOMBEO Dotación RNE-OS.100 : 220 l/hab/d Qm= ###### l/s
Población 162,016 habitantes CIUDAD BETA Dotación (Catastro) : Dotac. VARIABLE Pob. A,B y C Qm= ###### l/s
factor: 1 Caudales de Diseño: +15% PÉRDIDAS EN LA RED Qm= ###### l/s
K1= 1.273 Qd1= 0.000 l/s
K2= 2.401 Qd2= 0.000 l/s 781.46
DATOS PROPUESTO SIMILARES EN EL CUESTIONARIO-FICA 1439.34
Horas
Demanda parcial
(m3
)
00:00 - 01:00 54.00000
01:00 - 02:00 157.68000
02:00 - 03:00 816.48000
03:00 - 04:00 626.40000
04:00 - 05:00 648.00000
05:00 - 06:00 2,095.20000
06:00 - 07:00 2,872.80000
07:00 - 08:00 4,968.00000
08:00 - 09:00 5,248.80000
09:00 - 10:00 4,255.20000
10:00 - 11:00 3,386.88000
11:00 - 12:00 3,693.60000
12:00 - 13:00 8,538.13440
13:00 - 14:00 3,352.68000
14:00 - 15:00 2,419.20000
15:00 - 16:00 1,080.00000
16:00 - 17:00 777.60000
17:00 - 18:00 864.00000
18:00 - 19:00 1,231.20000
19:00 - 20:00 1,944.00000
20:00 - 21:00 1,900.80000
21:00 - 22:00 756.00000
22:00 - 23:00 82.08000
El valor de k1 sera mayor y k2 menor de efectuarse evaluaciones de campo. Los coeficientes de consumo horario con respecto al promedio serán menores con tendencia
central (picos menores)
El valor de k1 sera mayor y k2 menor de efectuarse evaluaciones de campo. Los coeficientes de consumo horario con respecto al promedio serán menores con tendencia
central (picos menores)
23:00 - 24:00 60.48000
ESCENARIO DE SISTEMA DE DISTRIBUCIÓN DE AGUA 2: BOMBEO- con 2 sub-escenarios
Para efecto de comparación se inicia el bombeo de caudales a las 06:00 horas:
Horas
Demanda parcial
(m3
)
Demanda
acumulada (m3
)
Tiempo parcial
(hora)
Tiempo
acumulado
(hora)
Caudal de
bombeo parcial -
24horas-
3bombas
(m3
/hora)
Caudal de
bombeo
acumulado-
24horas-
3bombas
(m3
/hora)
Volumen del
reservorio
Bombeo
24horas-
3bombas (m3
)
Caudal de
bombeo parcial -
8horas-
1bomba>capac.
(m3
/hora)
Caudal de
bombeo
acumulado-
8horas-
1bomba>capac.
(m3
/hora)
Volumen del
reservorio
Bombeo 8 horas -
1 bomba de >
capac. (m3
)
0.00000 0.00000 0.00000 0.00000 0.00000
06:00 - 07:00 2872.80000 2872.80000 1.00000 1.00000 2159.55060 2159.55060 -713.24940 6478.65180 6478.65180 3605.85180
07:00 - 08:00 4968.00000 7840.80000 1.00000 2.00000 2159.55060 4319.10120 -3521.69880 6478.65180 12957.30360 5116.50360
08:00 - 09:00 5248.80000 13089.60000 1.00000 3.00000 2159.55060 6478.65180 -6610.94820 6478.65180 19435.95540 6346.35540
09:00 - 10:00 4255.20000 17344.80000 1.00000 4.00000 2159.55060 8638.20240 -8706.59760 6478.65180 25914.60720 8569.80720
10:00 - 11:00 3386.88000 20731.68000 1.00000 5.00000 2159.55060 10797.75300 -9933.92700 25914.60720 5182.92720
11:00 - 12:00 3693.60000 24425.28000 1.00000 6.00000 2159.55060 12957.30360 -11467.97640 25914.60720 1489.32720
12:00 - 13:00 8538.13440 32963.41440 1.00000 7.00000 2159.55060 15116.85420 -17846.56020 25914.60720 -7048.80720
13:00 - 14:00 3352.68000 36316.09440 1.00000 8.00000 2159.55060 17276.40480 -19039.68960 25914.60720 -10401.48720
14:00 - 15:00 2419.20000 38735.29440 1.00000 9.00000 2159.55060 19435.95540 -19299.33900 6478.65180 32393.25900 -6342.03540
15:00 - 16:00 1080.00000 39815.29440 1.00000 10.00000 2159.55060 21595.50600 -18219.78840 6478.65180 38871.91080 -943.38360
16:00 - 17:00 777.60000 40592.89440 1.00000 11.00000 2159.55060 23755.05660 -16837.83780 6478.65180 45350.56260 4757.66820
17:00 - 18:00 864.00000 41456.89440 1.00000 12.00000 2159.55060 25914.60720 -15542.28720 6478.65180 51829.21440 10372.32000
18:00 - 19:00 1231.20000 42688.09440 1.00000 13.00000 2159.55060 28074.15780 -14613.93660 51829.21440 9141.12000
19:00 - 20:00 1944.00000 44632.09440 1.00000 14.00000 2159.55060 30233.70840 -14398.38600 51829.21440 7197.12000
20:00 - 21:00 1900.80000 46532.89440 1.00000 15.00000 2159.55060 32393.25900 -14139.63540 51829.21440 5296.32000
21:00 - 22:00 756.00000 47288.89440 1.00000 16.00000 2159.55060 34552.80960 -12736.08480 51829.21440 4540.32000
22:00 - 23:00 82.08000 47370.97440 1.00000 17.00000 2159.55060 36712.36020 -10658.61420 51829.21440 4458.24000
23:00 - 24:00 60.48000 47431.45440 1.00000 18.00000 2159.55060 38871.91080 -8559.54360 51829.21440 4397.76000
00:00 - 01:00 54.00000 47485.45440 1.00000 19.00000 2159.55060 41031.46140 -6453.99300 51829.21440 4343.76000
01:00 - 02:00 157.68000 47643.13440 1.00000 20.00000 2159.55060 43191.01200 -4452.12240 51829.21440 4186.08000
02:00 - 03:00 816.48000 48459.61440 1.00000 21.00000 2159.55060 45350.56260 -3109.05180 51829.21440 3369.60000
03:00 - 04:00 626.40000 49086.01440 1.00000 22.00000 2159.55060 47510.11320 -1575.90120 51829.21440 2743.20000
04:00 - 05:00 648.00000 49734.01440 1.00000 23.00000 2159.55060 49669.66380 -64.35060 51829.21440 2095.20000
05:00 - 06:00 2095.20000 51829.21440 1.00000 24.00000 2159.55060 51829.21440 0.00000 51829.21440 0.00000
90000
Título del gráfico
0
10000
20000
30000
40000
50000
60000
70000
80000
90000
06:00
-
07:00
07:00
-
08:00
08:00
-
09:00
09:00
-
10:00
10:00
-
11:00
11:00
-
12:00
12:00
-
13:00
13:00
-
14:00
14:00
-
15:00
15:00
-
16:00
16:00
-
17:00
17:00
-
18:00
18:00
-
19:00
19:00
-
20:00
20:00
-
21:00
21:00
-
22:00
22:00
-
23:00
23:00
-
24:00
00:00
-
01:00
01:00
-
02:00
02:00
-
03:00
03:00
-
04:00
04:00
-
05:00
05:00
-
06:00
m
3
Horas
Título del gráfico
Demanda acumulada (m3) Caudal de bombeo acumulado-24horas-3bombas (m3/hora)
Caudal de bombeo acumulado-8horas-1bomba>capac. (m3/hora)
30000
40000
Título del gráfico
PROCESO OPERATIVO-TÉCNICO-NORMATIVO PARA DETERMINAR EL VOLUMEN TOTAL DE RESERVORIOS EN POBLACIONES CON SOLO ACTIVIDADES DOMÉSTICAS
Q3bomba= Demanda acumulada en 24 horas / 24 horas = 51,829.2144 ÷ 24.0000 = 2,159.551 m
3
/hora
599.875 l/s
VRconsumo= 0.0000 - -19,299.3390 = 19,299.3390 m
3
19299339 litros
Horas de bombeo = 8 horas
Q1bomba= Demanda acumulada en 24 horas / 24 horas = 51,829.2144 ÷ 8.0000 = 6,478.652 m
3
/hora
1799.626 l/s
VRconsumo= 10,372.3200 - -10,401.4872 = 20,773.8072 m3
20773807.2 litros
RESPUESTA 01:
Capacidad de descarga de una (01) bomba al operar 8 horas discontinuas durante el día (de 06:00 a 10:00 horas y de 14:00-18:00 horas), para lograr cubrir
el total del volumen requerido por la demanda promedio de consumo o regulación diario de ciudad BETA es:
En caso de usar una (01) sola bomba por 8 horas al día de forma discontinua (4 horas cada etapa), la capacidad del reservorio para permitir a la bomba
cubrir de forma continua las 24 horas con la descarga requerida por la demanda de consumo de la ciudad BETA
Capacidad de descarga de tres (03) bombas simples, operando cada uno 8 horas cubriendo el suministro continuo de caudal las 24 horas del día para
satisfacer el total del volumen requerido por la demanda promedio de consumo diario de la ciudad BETA
En caso se usar tres (03) bombas simples operando 24 horas (8 horas c/u) la capacidad del reservorio para permitir a la bombas cubrir de forma continua las
24 horas con la descarga requerida por la demanda de consumo o regulación de la ciudad BETA es:
-20000
-10000
0
10000
20000
06:00
-
07:00
07:00
-
08:00
08:00
-
09:00
09:00
-
10:00
10:00
-
11:00
11:00
-
12:00
12:00
-
13:00
13:00
-
14:00
14:00
-
15:00
15:00
-
16:00
16:00
-
17:00
17:00
-
18:00
18:00
-
19:00
19:00
-
20:00
20:00
-
21:00
21:00
-
22:00
22:00
-
23:00
23:00
-
24:00
00:00
-
01:00
01:00
-
02:00
02:00
-
03:00
03:00
-
04:00
04:00
-
05:00
05:00
-
06:00
m
3
Horas
Volumen del reservorio Bombeo 24horas-3bombas (m3) Volumen del reservorio Bombeo 8 horas -1 bomba de > capac. (m3)
DIFER. DE VOLUMEN DE CONSUMO ESCENARIO (01) - (02) -1,474.4682 m
3
-1,474,468.20 litros
VRconsumo= 0.0000 m3
VRconsumo= 0.0000 NO EXISTE VOLUMEN DE AGUA 0 m3
DE 14:00 -06:00 horas NO HABRÁ AGUA EN EL RESERVORIO
RESPUESTA 02: DIECIOCHO (18) HORAS
DIFER. DE VOLUMEN DE CONSUMO ESCENARIO (01) - (02) 0.0000 m3
VRconsumo= 0.0000 m
3
05:00 - 06:00
01 HORA DE MAYOR RIESGO
VRconsumo= VRregulación= OS.030 RNE
VRconsumo= 19,299.3390 m
3
19,295 m
3
RECALCULAR o VRconsumo= 12,957.3036 m
3
Para poblaciones >10,000 hab., netamente destinada a viviendas (OS.100.RNE) NO SE CONSIDERA OBLIGATORIO (Inciso A Art. 1.6 - Norma OS.100)
VRincendio= 50 m3 (OS.030)
VRincendio= 0 m3 (OS.100)
Qi = 15 l/s 01 Grifo C.I.= 16 l/s
VRincendio= 216.000 m3 Horas = 2 horas = 7200 segundos
4 horas = 14400 segundos
VRreserva= 25% Vralmacenam. Previa justificación. Se recomienda usar 25 % del Volumen total de almacenamiento
Optamos por el proceso que nos suministre un mayor valor resultante (En caso de
existir compatibilidad con Vconsumo)
(Usando el valor mín. si 50 m3 no es concordante con
el Vcons.)
De optar por un bombeo con tres (03) bombas funcionando 8 horas cada uno para cubrir de forma continua las 24 horas del día, el volumen que contiene el
reservorio que satisfaga la demanda de consumo a las 06:00 horas es:
De optar por un bombeo con una sola bomba por 8 horas discontinuas (De 06:00 a 10:00horas y de 14:00 a 18:00horas), en que hora ocurre el riesgo mayor
de que el reservorio no tiene agua para cubrir la demanda de consumo por variaciones horarias, incendios y reserva para atender interrupciones, si se omite
De elegir un sistema con tres (03) bombas funcionando 8 horas cada una para cubrir 24 horas de suministro continuo, el volumen de diseño del reservorio
para cubrir a demanda de consumo por variaciones horarias, incendios y reserva para atender interrupciones, EN UNA ZONA NETAMENTE DESTINADA A
VIVIENDA es:
SE NECESITA UN RESERVORIO DE MAYOR VOLUMEN SE
REQUIERE ÁNALISIS DE RENTABILIDAD AL CONTAR CON UN
VOLUMEN PERMANENTE DE AGUA PARA CUBRIR VARIACIONES
HORARIAS DE CONSUMO
De optar por un bombeo con una sola bomba por 8 horas discontinuas, el volumen que contiene el reservorio para cubrir la demanda de consumo a las
06:00 horas es:
Norma OS.030:
Valmac. = Vregulac. + Vincendio + Vreserva
Vregulac. = Diagrama Masa ó 25% del Qprom. en 24 horas
Vreserva = 25% Vtotal
Vreserva = 33% (Vregulac. + Vincendio)
Vreserva = Qp x t ( 2 hr. < t > 4 hr. )
Vincendio = 50 m3 (para viviendas) ; GCIS con 3,000 m3 de volumen
aparente (Edif. de uso comercial o industrial)
VRalmacenam.= VRconsumo + VRincendio + VReserva =
19,299.3390 + 0.000 + 25% Vralmacenam.
VRalmacenam.= (VRconsumo + VRincendio) / 0.75 = 25,732.452 m
3
25,732,452 litros Vreserva = 6,433.1130 m
3
VOLUMEN TOTAL RESERV-ESCENARIO 01: 03 BOMBAS/24HR: 25,732.000 m
3
25.0%
VRconsumo= 20,773.8072 m
3
VRconsumo= VRregulación= OS.030 RNE
Para poblaciones >10,000 hab., netamente destinada a viviendas (OS.100.RNE) NO SE CONSIDERA OBLIGATORIO (Inciso A Art. 1.6 - Norma OS.100)
VRincendio= 50 m3 (OS.030)
VRincendio= 0 m3 (OS.100)
VRincendio= 0.000 m
3
Qi = 15 l/s 1 hidrante de 15 l/s cada uno
Horas = 2 horas = 7200 segundos
4 horas = 14400 segundos
VRreserva= 25% Vralmacenam. Previa justificación. Se recomienda usar 25 % del Volumen total de almacenamiento
VRalmacenam.= VRconsumo + VRincendio + VReserva =
20,773.8072 + 0.000 + 25% Vralmacenam.
VRalmacenam.= (VRconsumo + VRincendio) / 0.75 = 27,698.410 m
3
27,698,410 litros Vreserva = 6,924.6024 m
3
VOLUMEN TOTAL RESERV.-ESCENAR.02- 01 BOMBAS/08 HR: 27,698.000 m
3
25.0%
DIFERENCIA: 1,965.958 m
3
1,965,958 litros 1,965.958 m
3
PROCESO OPERATIVO-TÉCNICO-NORMATIVO PARA DETERMINAR EL VOLUMEN TOTAL DE RESERVORIOS EN POBLACIONES CON SOLO ACTIVIDADES DOMÉSTICAS
VRconsumo= 19,299.3390 m
3
VRconsumo= VRregulación= OS.030 RNE
Usando el gráfico para agua contra incendio de sólidos y las recomendaciones mínimas establecidas en la Norma OS.030 RNE:
Optamos por el proceso que nos suministre un mayor valor resultante (En caso de
existir compatibilidad con Vconsumo)
(Usando el valor mín. si 50 m3 no es concordante con
el Vcons.)
De optar por un bombeo con una (01) bomba funcionando 8 horas discontinuas de 4 horas (de 06:00 a 10:00 horas y de 14:00 a 18:00 horas) cada etapa, el
De optar por un bombeo con tres (03) bombas funcionando 8 horas cada una para cubrir 24 horas de suministro continuo, el volumen de diseño del
Volumen aparente de incendio mínimo = 3,000 m3
Factor de apilamiento intermedio (g)= 0.1
Para determinar el Volumen del reservorio contra incendio, del gráfico se obtiene:
Qincendio= 48 l/s Para extinguir el fuego
Qi = 48 l/s 3 hidrantes de 16 l/s cada uno
Horas = 2 horas = 7200 segundos
4 horas = 14400 segundos
VRincendio= 345.600 m3 (Usando el promedio Vol. Aparente y g=0.1 y tmín. 2 horas)
Para determinar el Volumen del reservorio para reserva, del gráfico se obtiene:
VRreserva= 145 m3
VRalmacenam.= VRconsumo + VRincendio + VReserva =
19,299.3390 + 345.600 + 145
VRalmacenam.= 19,789.939 m3 19,789,939 litros
DIFERENCIA: -5,942.513 5,942,513 litros 5,942.513 m
3
VRconsumo= 20,773.8072 m3 VRconsumo= VRregulación= OS.030 RNE
Usando el gráfico para agua contra incendio de sólidos y las recomendaciones mínimas establecidas en la Norma OS.030 RNE:
Volumen aparente de incendio mínimo = 3,000 m3
Factor de apilamiento intermedio (g)= 0.5
Para determinar el Volumen del reservorio contra incendio, del gráfico se obtiene:
Qincendio= 48 l/s Para extinguir el fuego
Qi = 48 l/s 4 hidrantes de 16 l/s cada uno
Horas = 2 horas = 7200 segundos
4 horas = 14400 segundos
VRincendio= 345.600 m3 (Usando el promedio Vol. Aparente y g=0.1 y tmín. 2 horas)
Para determinar el Volumen del reservorio para reserva, del gráfico se obtiene:
VRreserva= 145 m3
VRalmacenam.= VRconsumo + VRincendio + VReserva =
20,773.8072 + 345.600 + 145
VRalmacenam.= 21,264.407 m3 21,264,407 litros
RESPUESTA 03: DIFERENCIA: 1,474.468 m3 1,474,468 litros 1,474.468 m
3
DIFERENCIAS ENTRE ZONA DE VIVIEND. DE COMERC.INDUST. -6,434.002 m3
De optar por un bombeo con una (01) bomba funcionando 8 horas discontinuas de 4 horas cada etapa (de 06:00 a 10:00 horas y de 14:00 a 18:00 horas), el

Más contenido relacionado

Similar a SOLUCIONARIO 1 EVSUMATIVA 2 AAA 2020

SOLUCIONARIO 3 EVSUMATIVA 1 AAA 2020 UNHEVAL FICA PAUCAR
SOLUCIONARIO 3 EVSUMATIVA 1 AAA 2020 UNHEVAL FICA PAUCARSOLUCIONARIO 3 EVSUMATIVA 1 AAA 2020 UNHEVAL FICA PAUCAR
SOLUCIONARIO 3 EVSUMATIVA 1 AAA 2020 UNHEVAL FICA PAUCARRODOLFOLUIS8
 
Capacitacion_CIERRE-- F9--2021_05_27.pdf
Capacitacion_CIERRE-- F9--2021_05_27.pdfCapacitacion_CIERRE-- F9--2021_05_27.pdf
Capacitacion_CIERRE-- F9--2021_05_27.pdfeconleninalvarado01
 
02. Presentacion_SCG_FITA 2020-2021 (1).pptx
02. Presentacion_SCG_FITA 2020-2021 (1).pptx02. Presentacion_SCG_FITA 2020-2021 (1).pptx
02. Presentacion_SCG_FITA 2020-2021 (1).pptxTeobaldo Pozo Jabo
 
Situación Fiscal 2018 - Junio 2019 Municipio de San José de Cúcuta
Situación Fiscal 2018 - Junio 2019 Municipio de San José de CúcutaSituación Fiscal 2018 - Junio 2019 Municipio de San José de Cúcuta
Situación Fiscal 2018 - Junio 2019 Municipio de San José de CúcutaDAF MHCP
 
Presentacion escenarios economicos 2020 CLAVES
Presentacion escenarios economicos 2020 CLAVESPresentacion escenarios economicos 2020 CLAVES
Presentacion escenarios economicos 2020 CLAVESNelson Perez Alonso
 
Todos los datos del Presupuesto 2018 de Misiones
Todos los datos del Presupuesto 2018 de MisionesTodos los datos del Presupuesto 2018 de Misiones
Todos los datos del Presupuesto 2018 de MisionesEconomis
 
Situación Fiscal 2018 - Junio 2019 Municipio de Sincelejo
Situación Fiscal 2018 - Junio 2019 Municipio de SincelejoSituación Fiscal 2018 - Junio 2019 Municipio de Sincelejo
Situación Fiscal 2018 - Junio 2019 Municipio de SincelejoDAF MHCP
 
Memoria de Cálculo - Salaverry HASTA ALCANTARILLADO.pdf
Memoria de Cálculo  - Salaverry HASTA ALCANTARILLADO.pdfMemoria de Cálculo  - Salaverry HASTA ALCANTARILLADO.pdf
Memoria de Cálculo - Salaverry HASTA ALCANTARILLADO.pdfThaliaGuillen2
 
El ingreso medio per cápita familiar es de $32.934 en La Rioja
El ingreso medio per cápita familiar es de $32.934 en La RiojaEl ingreso medio per cápita familiar es de $32.934 en La Rioja
El ingreso medio per cápita familiar es de $32.934 en La RiojaEduardo Nelson German
 
Inversion Publicitaria Argentina 2009
Inversion Publicitaria Argentina 2009Inversion Publicitaria Argentina 2009
Inversion Publicitaria Argentina 2009mediosyempresas.com
 
Inversion Publicitaria Argentina 2009.-
Inversion Publicitaria Argentina 2009.-Inversion Publicitaria Argentina 2009.-
Inversion Publicitaria Argentina 2009.-guest1c6b534
 
Foro-Piura-2022-Javier-Bereche-Contexto-economico-y-oportunidades-de-inversio...
Foro-Piura-2022-Javier-Bereche-Contexto-economico-y-oportunidades-de-inversio...Foro-Piura-2022-Javier-Bereche-Contexto-economico-y-oportunidades-de-inversio...
Foro-Piura-2022-Javier-Bereche-Contexto-economico-y-oportunidades-de-inversio...AbigailRamirez92
 
sistema de agua y alcantarillado.1.docx
sistema de agua y alcantarillado.1.docxsistema de agua y alcantarillado.1.docx
sistema de agua y alcantarillado.1.docxCueLLiDurO
 

Similar a SOLUCIONARIO 1 EVSUMATIVA 2 AAA 2020 (20)

SOLUCIONARIO 3 EVSUMATIVA 1 AAA 2020 UNHEVAL FICA PAUCAR
SOLUCIONARIO 3 EVSUMATIVA 1 AAA 2020 UNHEVAL FICA PAUCARSOLUCIONARIO 3 EVSUMATIVA 1 AAA 2020 UNHEVAL FICA PAUCAR
SOLUCIONARIO 3 EVSUMATIVA 1 AAA 2020 UNHEVAL FICA PAUCAR
 
Presentacion_DGPIP_2023.pptx
Presentacion_DGPIP_2023.pptxPresentacion_DGPIP_2023.pptx
Presentacion_DGPIP_2023.pptx
 
Capacitacion_CIERRE-- F9--2021_05_27.pdf
Capacitacion_CIERRE-- F9--2021_05_27.pdfCapacitacion_CIERRE-- F9--2021_05_27.pdf
Capacitacion_CIERRE-- F9--2021_05_27.pdf
 
02. Presentacion_SCG_FITA 2020-2021 (1).pptx
02. Presentacion_SCG_FITA 2020-2021 (1).pptx02. Presentacion_SCG_FITA 2020-2021 (1).pptx
02. Presentacion_SCG_FITA 2020-2021 (1).pptx
 
Situación Fiscal 2018 - Junio 2019 Municipio de San José de Cúcuta
Situación Fiscal 2018 - Junio 2019 Municipio de San José de CúcutaSituación Fiscal 2018 - Junio 2019 Municipio de San José de Cúcuta
Situación Fiscal 2018 - Junio 2019 Municipio de San José de Cúcuta
 
Proyecto relleno sanitario las cuadras
Proyecto relleno sanitario las cuadrasProyecto relleno sanitario las cuadras
Proyecto relleno sanitario las cuadras
 
COYUNTURA ECONOMICA C.A.B.A.
COYUNTURA ECONOMICA C.A.B.A.COYUNTURA ECONOMICA C.A.B.A.
COYUNTURA ECONOMICA C.A.B.A.
 
Presentacion escenarios economicos 2020 CLAVES
Presentacion escenarios economicos 2020 CLAVESPresentacion escenarios economicos 2020 CLAVES
Presentacion escenarios economicos 2020 CLAVES
 
Todos los datos del Presupuesto 2018 de Misiones
Todos los datos del Presupuesto 2018 de MisionesTodos los datos del Presupuesto 2018 de Misiones
Todos los datos del Presupuesto 2018 de Misiones
 
Situación Fiscal 2018 - Junio 2019 Municipio de Sincelejo
Situación Fiscal 2018 - Junio 2019 Municipio de SincelejoSituación Fiscal 2018 - Junio 2019 Municipio de Sincelejo
Situación Fiscal 2018 - Junio 2019 Municipio de Sincelejo
 
Memoria de Cálculo - Salaverry HASTA ALCANTARILLADO.pdf
Memoria de Cálculo  - Salaverry HASTA ALCANTARILLADO.pdfMemoria de Cálculo  - Salaverry HASTA ALCANTARILLADO.pdf
Memoria de Cálculo - Salaverry HASTA ALCANTARILLADO.pdf
 
El ingreso medio per cápita familiar es de $32.934 en La Rioja
El ingreso medio per cápita familiar es de $32.934 en La RiojaEl ingreso medio per cápita familiar es de $32.934 en La Rioja
El ingreso medio per cápita familiar es de $32.934 en La Rioja
 
Presupuestos Junta
Presupuestos Junta Presupuestos Junta
Presupuestos Junta
 
Inversion Publicitaria Argentina 2009
Inversion Publicitaria Argentina 2009Inversion Publicitaria Argentina 2009
Inversion Publicitaria Argentina 2009
 
Inversion Publicitaria Argentina 2009.-
Inversion Publicitaria Argentina 2009.-Inversion Publicitaria Argentina 2009.-
Inversion Publicitaria Argentina 2009.-
 
Gustavo Adolfo Marulanda – Director Catastro Bogotá
Gustavo Adolfo Marulanda – Director Catastro BogotáGustavo Adolfo Marulanda – Director Catastro Bogotá
Gustavo Adolfo Marulanda – Director Catastro Bogotá
 
Foro-Piura-2022-Javier-Bereche-Contexto-economico-y-oportunidades-de-inversio...
Foro-Piura-2022-Javier-Bereche-Contexto-economico-y-oportunidades-de-inversio...Foro-Piura-2022-Javier-Bereche-Contexto-economico-y-oportunidades-de-inversio...
Foro-Piura-2022-Javier-Bereche-Contexto-economico-y-oportunidades-de-inversio...
 
Evaluacion economica
Evaluacion economicaEvaluacion economica
Evaluacion economica
 
sistema de agua y alcantarillado.1.docx
sistema de agua y alcantarillado.1.docxsistema de agua y alcantarillado.1.docx
sistema de agua y alcantarillado.1.docx
 
04 estudio de_mercado (1)
04 estudio de_mercado (1)04 estudio de_mercado (1)
04 estudio de_mercado (1)
 

Más de RODOLFOLUIS8

Expo diseno de_losas_y_vigas
Expo diseno de_losas_y_vigasExpo diseno de_losas_y_vigas
Expo diseno de_losas_y_vigasRODOLFOLUIS8
 
Capitulo 21.1 RNE E060 CONCRETO ARMADO
Capitulo 21.1 RNE E060 CONCRETO ARMADOCapitulo 21.1 RNE E060 CONCRETO ARMADO
Capitulo 21.1 RNE E060 CONCRETO ARMADORODOLFOLUIS8
 
HIDROLOGIA CON ARCGIS UNHEVAL FICA DOH
HIDROLOGIA CON ARCGIS UNHEVAL FICA DOH HIDROLOGIA CON ARCGIS UNHEVAL FICA DOH
HIDROLOGIA CON ARCGIS UNHEVAL FICA DOH RODOLFOLUIS8
 
TRABAJO ESCALONADO 7 UNHEVAL FICA DOH PAUCAR
TRABAJO ESCALONADO 7 UNHEVAL FICA DOH PAUCARTRABAJO ESCALONADO 7 UNHEVAL FICA DOH PAUCAR
TRABAJO ESCALONADO 7 UNHEVAL FICA DOH PAUCARRODOLFOLUIS8
 
TRABAJO ESCALONADO 6 UNHEVAL FICA DOH PAUCAR
TRABAJO ESCALONADO 6 UNHEVAL FICA DOH PAUCARTRABAJO ESCALONADO 6 UNHEVAL FICA DOH PAUCAR
TRABAJO ESCALONADO 6 UNHEVAL FICA DOH PAUCARRODOLFOLUIS8
 
TRABAJO ESCALONADO 5 UNHEVAL FICA DOH PAUCAR
TRABAJO ESCALONADO 5 UNHEVAL FICA DOH PAUCARTRABAJO ESCALONADO 5 UNHEVAL FICA DOH PAUCAR
TRABAJO ESCALONADO 5 UNHEVAL FICA DOH PAUCARRODOLFOLUIS8
 
Info5 GEOTECNIA 2 EMPUJE DE TIERRAS
Info5 GEOTECNIA 2 EMPUJE DE TIERRASInfo5 GEOTECNIA 2 EMPUJE DE TIERRAS
Info5 GEOTECNIA 2 EMPUJE DE TIERRASRODOLFOLUIS8
 
Info4 geotecnia 2 EL EMPUJE DE TIERRAS
Info4 geotecnia 2 EL EMPUJE DE TIERRAS Info4 geotecnia 2 EL EMPUJE DE TIERRAS
Info4 geotecnia 2 EL EMPUJE DE TIERRAS RODOLFOLUIS8
 
SOLUCIONARIO 2 EVSUMATIVA 1 AAA 2020 UNHEVAL FICA PAUCAR
 SOLUCIONARIO 2 EVSUMATIVA 1 AAA 2020 UNHEVAL FICA PAUCAR SOLUCIONARIO 2 EVSUMATIVA 1 AAA 2020 UNHEVAL FICA PAUCAR
SOLUCIONARIO 2 EVSUMATIVA 1 AAA 2020 UNHEVAL FICA PAUCARRODOLFOLUIS8
 
SOLUCIONARIO 1 EVSUMATIVA 1 AAA 2020 UNHEVAL FICA PAUCAR
SOLUCIONARIO 1 EVSUMATIVA 1 AAA 2020 UNHEVAL FICA PAUCARSOLUCIONARIO 1 EVSUMATIVA 1 AAA 2020 UNHEVAL FICA PAUCAR
SOLUCIONARIO 1 EVSUMATIVA 1 AAA 2020 UNHEVAL FICA PAUCARRODOLFOLUIS8
 
EVALUACIÓN SUMATIVA 01-AAA2020.2g1-CPyM-FICA-UNHEVAL
EVALUACIÓN SUMATIVA 01-AAA2020.2g1-CPyM-FICA-UNHEVALEVALUACIÓN SUMATIVA 01-AAA2020.2g1-CPyM-FICA-UNHEVAL
EVALUACIÓN SUMATIVA 01-AAA2020.2g1-CPyM-FICA-UNHEVALRODOLFOLUIS8
 
SOLUCIONARIO 3 EVSUMATIVA 2 AAA 2020 UNHEVAL FICA PAUCAR
SOLUCIONARIO 3 EVSUMATIVA 2 AAA 2020 UNHEVAL FICA PAUCARSOLUCIONARIO 3 EVSUMATIVA 2 AAA 2020 UNHEVAL FICA PAUCAR
SOLUCIONARIO 3 EVSUMATIVA 2 AAA 2020 UNHEVAL FICA PAUCARRODOLFOLUIS8
 
SOLUCIONARIO 2 EVSUMATIVA 2 AAA 2020 UNHEVAL FICA PAUCAR
SOLUCIONARIO 2 EVSUMATIVA 2 AAA 2020 UNHEVAL FICA PAUCARSOLUCIONARIO 2 EVSUMATIVA 2 AAA 2020 UNHEVAL FICA PAUCAR
SOLUCIONARIO 2 EVSUMATIVA 2 AAA 2020 UNHEVAL FICA PAUCARRODOLFOLUIS8
 
RESPEUSTAS EVASUMATIVA2 HUANUCO FICA UNHEVAL AAA PAUCAR
RESPEUSTAS EVASUMATIVA2 HUANUCO FICA UNHEVAL AAA PAUCARRESPEUSTAS EVASUMATIVA2 HUANUCO FICA UNHEVAL AAA PAUCAR
RESPEUSTAS EVASUMATIVA2 HUANUCO FICA UNHEVAL AAA PAUCARRODOLFOLUIS8
 
EVSUMATIVA 2 HUANUCO UNHEVAL FICA AAA PAUCAR
EVSUMATIVA 2 HUANUCO UNHEVAL FICA AAA PAUCAREVSUMATIVA 2 HUANUCO UNHEVAL FICA AAA PAUCAR
EVSUMATIVA 2 HUANUCO UNHEVAL FICA AAA PAUCARRODOLFOLUIS8
 
TRABAJO ESCALONADO 10 HUANUCO UNHEVAL FICA AAA PAUCAR
TRABAJO ESCALONADO 10 HUANUCO UNHEVAL FICA AAA PAUCARTRABAJO ESCALONADO 10 HUANUCO UNHEVAL FICA AAA PAUCAR
TRABAJO ESCALONADO 10 HUANUCO UNHEVAL FICA AAA PAUCARRODOLFOLUIS8
 
TRABAJO ESCALONADO 9 HUANUCO UNHEVAL FICA AAA PAUCAR
TRABAJO ESCALONADO 9 HUANUCO UNHEVAL FICA AAA PAUCARTRABAJO ESCALONADO 9 HUANUCO UNHEVAL FICA AAA PAUCAR
TRABAJO ESCALONADO 9 HUANUCO UNHEVAL FICA AAA PAUCARRODOLFOLUIS8
 
TRABAJO ESCALONADO 7 HUANUCO UNHEVAL FICA AAA PAUCAR
TRABAJO ESCALONADO 7 HUANUCO UNHEVAL FICA AAA PAUCARTRABAJO ESCALONADO 7 HUANUCO UNHEVAL FICA AAA PAUCAR
TRABAJO ESCALONADO 7 HUANUCO UNHEVAL FICA AAA PAUCARRODOLFOLUIS8
 
TRABAJO ESCALONADO 6 HUANUCO UNHEVAL FICA AAA PAUCAR
TRABAJO ESCALONADO 6 HUANUCO UNHEVAL FICA AAA PAUCARTRABAJO ESCALONADO 6 HUANUCO UNHEVAL FICA AAA PAUCAR
TRABAJO ESCALONADO 6 HUANUCO UNHEVAL FICA AAA PAUCARRODOLFOLUIS8
 
TRABAJO ESCALONADO 5 HUANUCO UNHEVAL FICA AAA PAUCAR
TRABAJO ESCALONADO 5 HUANUCO UNHEVAL FICA AAA PAUCARTRABAJO ESCALONADO 5 HUANUCO UNHEVAL FICA AAA PAUCAR
TRABAJO ESCALONADO 5 HUANUCO UNHEVAL FICA AAA PAUCARRODOLFOLUIS8
 

Más de RODOLFOLUIS8 (20)

Expo diseno de_losas_y_vigas
Expo diseno de_losas_y_vigasExpo diseno de_losas_y_vigas
Expo diseno de_losas_y_vigas
 
Capitulo 21.1 RNE E060 CONCRETO ARMADO
Capitulo 21.1 RNE E060 CONCRETO ARMADOCapitulo 21.1 RNE E060 CONCRETO ARMADO
Capitulo 21.1 RNE E060 CONCRETO ARMADO
 
HIDROLOGIA CON ARCGIS UNHEVAL FICA DOH
HIDROLOGIA CON ARCGIS UNHEVAL FICA DOH HIDROLOGIA CON ARCGIS UNHEVAL FICA DOH
HIDROLOGIA CON ARCGIS UNHEVAL FICA DOH
 
TRABAJO ESCALONADO 7 UNHEVAL FICA DOH PAUCAR
TRABAJO ESCALONADO 7 UNHEVAL FICA DOH PAUCARTRABAJO ESCALONADO 7 UNHEVAL FICA DOH PAUCAR
TRABAJO ESCALONADO 7 UNHEVAL FICA DOH PAUCAR
 
TRABAJO ESCALONADO 6 UNHEVAL FICA DOH PAUCAR
TRABAJO ESCALONADO 6 UNHEVAL FICA DOH PAUCARTRABAJO ESCALONADO 6 UNHEVAL FICA DOH PAUCAR
TRABAJO ESCALONADO 6 UNHEVAL FICA DOH PAUCAR
 
TRABAJO ESCALONADO 5 UNHEVAL FICA DOH PAUCAR
TRABAJO ESCALONADO 5 UNHEVAL FICA DOH PAUCARTRABAJO ESCALONADO 5 UNHEVAL FICA DOH PAUCAR
TRABAJO ESCALONADO 5 UNHEVAL FICA DOH PAUCAR
 
Info5 GEOTECNIA 2 EMPUJE DE TIERRAS
Info5 GEOTECNIA 2 EMPUJE DE TIERRASInfo5 GEOTECNIA 2 EMPUJE DE TIERRAS
Info5 GEOTECNIA 2 EMPUJE DE TIERRAS
 
Info4 geotecnia 2 EL EMPUJE DE TIERRAS
Info4 geotecnia 2 EL EMPUJE DE TIERRAS Info4 geotecnia 2 EL EMPUJE DE TIERRAS
Info4 geotecnia 2 EL EMPUJE DE TIERRAS
 
SOLUCIONARIO 2 EVSUMATIVA 1 AAA 2020 UNHEVAL FICA PAUCAR
 SOLUCIONARIO 2 EVSUMATIVA 1 AAA 2020 UNHEVAL FICA PAUCAR SOLUCIONARIO 2 EVSUMATIVA 1 AAA 2020 UNHEVAL FICA PAUCAR
SOLUCIONARIO 2 EVSUMATIVA 1 AAA 2020 UNHEVAL FICA PAUCAR
 
SOLUCIONARIO 1 EVSUMATIVA 1 AAA 2020 UNHEVAL FICA PAUCAR
SOLUCIONARIO 1 EVSUMATIVA 1 AAA 2020 UNHEVAL FICA PAUCARSOLUCIONARIO 1 EVSUMATIVA 1 AAA 2020 UNHEVAL FICA PAUCAR
SOLUCIONARIO 1 EVSUMATIVA 1 AAA 2020 UNHEVAL FICA PAUCAR
 
EVALUACIÓN SUMATIVA 01-AAA2020.2g1-CPyM-FICA-UNHEVAL
EVALUACIÓN SUMATIVA 01-AAA2020.2g1-CPyM-FICA-UNHEVALEVALUACIÓN SUMATIVA 01-AAA2020.2g1-CPyM-FICA-UNHEVAL
EVALUACIÓN SUMATIVA 01-AAA2020.2g1-CPyM-FICA-UNHEVAL
 
SOLUCIONARIO 3 EVSUMATIVA 2 AAA 2020 UNHEVAL FICA PAUCAR
SOLUCIONARIO 3 EVSUMATIVA 2 AAA 2020 UNHEVAL FICA PAUCARSOLUCIONARIO 3 EVSUMATIVA 2 AAA 2020 UNHEVAL FICA PAUCAR
SOLUCIONARIO 3 EVSUMATIVA 2 AAA 2020 UNHEVAL FICA PAUCAR
 
SOLUCIONARIO 2 EVSUMATIVA 2 AAA 2020 UNHEVAL FICA PAUCAR
SOLUCIONARIO 2 EVSUMATIVA 2 AAA 2020 UNHEVAL FICA PAUCARSOLUCIONARIO 2 EVSUMATIVA 2 AAA 2020 UNHEVAL FICA PAUCAR
SOLUCIONARIO 2 EVSUMATIVA 2 AAA 2020 UNHEVAL FICA PAUCAR
 
RESPEUSTAS EVASUMATIVA2 HUANUCO FICA UNHEVAL AAA PAUCAR
RESPEUSTAS EVASUMATIVA2 HUANUCO FICA UNHEVAL AAA PAUCARRESPEUSTAS EVASUMATIVA2 HUANUCO FICA UNHEVAL AAA PAUCAR
RESPEUSTAS EVASUMATIVA2 HUANUCO FICA UNHEVAL AAA PAUCAR
 
EVSUMATIVA 2 HUANUCO UNHEVAL FICA AAA PAUCAR
EVSUMATIVA 2 HUANUCO UNHEVAL FICA AAA PAUCAREVSUMATIVA 2 HUANUCO UNHEVAL FICA AAA PAUCAR
EVSUMATIVA 2 HUANUCO UNHEVAL FICA AAA PAUCAR
 
TRABAJO ESCALONADO 10 HUANUCO UNHEVAL FICA AAA PAUCAR
TRABAJO ESCALONADO 10 HUANUCO UNHEVAL FICA AAA PAUCARTRABAJO ESCALONADO 10 HUANUCO UNHEVAL FICA AAA PAUCAR
TRABAJO ESCALONADO 10 HUANUCO UNHEVAL FICA AAA PAUCAR
 
TRABAJO ESCALONADO 9 HUANUCO UNHEVAL FICA AAA PAUCAR
TRABAJO ESCALONADO 9 HUANUCO UNHEVAL FICA AAA PAUCARTRABAJO ESCALONADO 9 HUANUCO UNHEVAL FICA AAA PAUCAR
TRABAJO ESCALONADO 9 HUANUCO UNHEVAL FICA AAA PAUCAR
 
TRABAJO ESCALONADO 7 HUANUCO UNHEVAL FICA AAA PAUCAR
TRABAJO ESCALONADO 7 HUANUCO UNHEVAL FICA AAA PAUCARTRABAJO ESCALONADO 7 HUANUCO UNHEVAL FICA AAA PAUCAR
TRABAJO ESCALONADO 7 HUANUCO UNHEVAL FICA AAA PAUCAR
 
TRABAJO ESCALONADO 6 HUANUCO UNHEVAL FICA AAA PAUCAR
TRABAJO ESCALONADO 6 HUANUCO UNHEVAL FICA AAA PAUCARTRABAJO ESCALONADO 6 HUANUCO UNHEVAL FICA AAA PAUCAR
TRABAJO ESCALONADO 6 HUANUCO UNHEVAL FICA AAA PAUCAR
 
TRABAJO ESCALONADO 5 HUANUCO UNHEVAL FICA AAA PAUCAR
TRABAJO ESCALONADO 5 HUANUCO UNHEVAL FICA AAA PAUCARTRABAJO ESCALONADO 5 HUANUCO UNHEVAL FICA AAA PAUCAR
TRABAJO ESCALONADO 5 HUANUCO UNHEVAL FICA AAA PAUCAR
 

Último

Sesión 02 TIPOS DE VALORIZACIONES CURSO Cersa
Sesión 02 TIPOS DE VALORIZACIONES CURSO CersaSesión 02 TIPOS DE VALORIZACIONES CURSO Cersa
Sesión 02 TIPOS DE VALORIZACIONES CURSO CersaXimenaFallaLecca1
 
hitos del desarrollo psicomotor en niños.docx
hitos del desarrollo psicomotor en niños.docxhitos del desarrollo psicomotor en niños.docx
hitos del desarrollo psicomotor en niños.docxMarcelaArancibiaRojo
 
Obras paralizadas en el sector construcción
Obras paralizadas en el sector construcciónObras paralizadas en el sector construcción
Obras paralizadas en el sector construcciónXimenaFallaLecca1
 
Condensadores de la rama de electricidad y magnetismo
Condensadores de la rama de electricidad y magnetismoCondensadores de la rama de electricidad y magnetismo
Condensadores de la rama de electricidad y magnetismosaultorressep
 
CHARLA DE INDUCCIÓN SEGURIDAD Y SALUD OCUPACIONAL
CHARLA DE INDUCCIÓN SEGURIDAD Y SALUD OCUPACIONALCHARLA DE INDUCCIÓN SEGURIDAD Y SALUD OCUPACIONAL
CHARLA DE INDUCCIÓN SEGURIDAD Y SALUD OCUPACIONALKATHIAMILAGRITOSSANC
 
Residente de obra y sus funciones que realiza .pdf
Residente de obra y sus funciones que realiza  .pdfResidente de obra y sus funciones que realiza  .pdf
Residente de obra y sus funciones que realiza .pdfevin1703e
 
presentacion medidas de seguridad riesgo eléctrico
presentacion medidas de seguridad riesgo eléctricopresentacion medidas de seguridad riesgo eléctrico
presentacion medidas de seguridad riesgo eléctricoalexcala5
 
01 MATERIALES AERONAUTICOS VARIOS clase 1.ppt
01 MATERIALES AERONAUTICOS VARIOS clase 1.ppt01 MATERIALES AERONAUTICOS VARIOS clase 1.ppt
01 MATERIALES AERONAUTICOS VARIOS clase 1.pptoscarvielma45
 
ECONOMIA APLICADA SEMANA 555555555544.pdf
ECONOMIA APLICADA SEMANA 555555555544.pdfECONOMIA APLICADA SEMANA 555555555544.pdf
ECONOMIA APLICADA SEMANA 555555555544.pdfmatepura
 
desarrollodeproyectoss inge. industrial
desarrollodeproyectoss  inge. industrialdesarrollodeproyectoss  inge. industrial
desarrollodeproyectoss inge. industrialGibranDiaz7
 
Seleccion de Fusibles en media tension fusibles
Seleccion de Fusibles en media tension fusiblesSeleccion de Fusibles en media tension fusibles
Seleccion de Fusibles en media tension fusiblesSaulSantiago25
 
Principales aportes de la carrera de William Edwards Deming
Principales aportes de la carrera de William Edwards DemingPrincipales aportes de la carrera de William Edwards Deming
Principales aportes de la carrera de William Edwards DemingKevinCabrera96
 
Controladores Lógicos Programables Usos y Ventajas
Controladores Lógicos Programables Usos y VentajasControladores Lógicos Programables Usos y Ventajas
Controladores Lógicos Programables Usos y Ventajasjuanprv
 
UNIDAD 3 ELECTRODOS.pptx para biopotenciales
UNIDAD 3 ELECTRODOS.pptx para biopotencialesUNIDAD 3 ELECTRODOS.pptx para biopotenciales
UNIDAD 3 ELECTRODOS.pptx para biopotencialesElianaCceresTorrico
 
Una estrategia de seguridad en la nube alineada al NIST
Una estrategia de seguridad en la nube alineada al NISTUna estrategia de seguridad en la nube alineada al NIST
Una estrategia de seguridad en la nube alineada al NISTFundación YOD YOD
 
Curso intensivo de soldadura electrónica en pdf
Curso intensivo de soldadura electrónica  en pdfCurso intensivo de soldadura electrónica  en pdf
Curso intensivo de soldadura electrónica en pdfFernandaGarca788912
 
¿QUE SON LOS AGENTES FISICOS Y QUE CUIDADOS TENER.pptx
¿QUE SON LOS AGENTES FISICOS Y QUE CUIDADOS TENER.pptx¿QUE SON LOS AGENTES FISICOS Y QUE CUIDADOS TENER.pptx
¿QUE SON LOS AGENTES FISICOS Y QUE CUIDADOS TENER.pptxguillermosantana15
 
COMPEDIOS ESTADISTICOS DE PERU EN EL 2023
COMPEDIOS ESTADISTICOS DE PERU EN EL 2023COMPEDIOS ESTADISTICOS DE PERU EN EL 2023
COMPEDIOS ESTADISTICOS DE PERU EN EL 2023RonaldoPaucarMontes
 
Calavera calculo de estructuras de cimentacion.pdf
Calavera calculo de estructuras de cimentacion.pdfCalavera calculo de estructuras de cimentacion.pdf
Calavera calculo de estructuras de cimentacion.pdfyoseka196
 
Elaboración de la estructura del ADN y ARN en papel.pdf
Elaboración de la estructura del ADN y ARN en papel.pdfElaboración de la estructura del ADN y ARN en papel.pdf
Elaboración de la estructura del ADN y ARN en papel.pdfKEVINYOICIAQUINOSORI
 

Último (20)

Sesión 02 TIPOS DE VALORIZACIONES CURSO Cersa
Sesión 02 TIPOS DE VALORIZACIONES CURSO CersaSesión 02 TIPOS DE VALORIZACIONES CURSO Cersa
Sesión 02 TIPOS DE VALORIZACIONES CURSO Cersa
 
hitos del desarrollo psicomotor en niños.docx
hitos del desarrollo psicomotor en niños.docxhitos del desarrollo psicomotor en niños.docx
hitos del desarrollo psicomotor en niños.docx
 
Obras paralizadas en el sector construcción
Obras paralizadas en el sector construcciónObras paralizadas en el sector construcción
Obras paralizadas en el sector construcción
 
Condensadores de la rama de electricidad y magnetismo
Condensadores de la rama de electricidad y magnetismoCondensadores de la rama de electricidad y magnetismo
Condensadores de la rama de electricidad y magnetismo
 
CHARLA DE INDUCCIÓN SEGURIDAD Y SALUD OCUPACIONAL
CHARLA DE INDUCCIÓN SEGURIDAD Y SALUD OCUPACIONALCHARLA DE INDUCCIÓN SEGURIDAD Y SALUD OCUPACIONAL
CHARLA DE INDUCCIÓN SEGURIDAD Y SALUD OCUPACIONAL
 
Residente de obra y sus funciones que realiza .pdf
Residente de obra y sus funciones que realiza  .pdfResidente de obra y sus funciones que realiza  .pdf
Residente de obra y sus funciones que realiza .pdf
 
presentacion medidas de seguridad riesgo eléctrico
presentacion medidas de seguridad riesgo eléctricopresentacion medidas de seguridad riesgo eléctrico
presentacion medidas de seguridad riesgo eléctrico
 
01 MATERIALES AERONAUTICOS VARIOS clase 1.ppt
01 MATERIALES AERONAUTICOS VARIOS clase 1.ppt01 MATERIALES AERONAUTICOS VARIOS clase 1.ppt
01 MATERIALES AERONAUTICOS VARIOS clase 1.ppt
 
ECONOMIA APLICADA SEMANA 555555555544.pdf
ECONOMIA APLICADA SEMANA 555555555544.pdfECONOMIA APLICADA SEMANA 555555555544.pdf
ECONOMIA APLICADA SEMANA 555555555544.pdf
 
desarrollodeproyectoss inge. industrial
desarrollodeproyectoss  inge. industrialdesarrollodeproyectoss  inge. industrial
desarrollodeproyectoss inge. industrial
 
Seleccion de Fusibles en media tension fusibles
Seleccion de Fusibles en media tension fusiblesSeleccion de Fusibles en media tension fusibles
Seleccion de Fusibles en media tension fusibles
 
Principales aportes de la carrera de William Edwards Deming
Principales aportes de la carrera de William Edwards DemingPrincipales aportes de la carrera de William Edwards Deming
Principales aportes de la carrera de William Edwards Deming
 
Controladores Lógicos Programables Usos y Ventajas
Controladores Lógicos Programables Usos y VentajasControladores Lógicos Programables Usos y Ventajas
Controladores Lógicos Programables Usos y Ventajas
 
UNIDAD 3 ELECTRODOS.pptx para biopotenciales
UNIDAD 3 ELECTRODOS.pptx para biopotencialesUNIDAD 3 ELECTRODOS.pptx para biopotenciales
UNIDAD 3 ELECTRODOS.pptx para biopotenciales
 
Una estrategia de seguridad en la nube alineada al NIST
Una estrategia de seguridad en la nube alineada al NISTUna estrategia de seguridad en la nube alineada al NIST
Una estrategia de seguridad en la nube alineada al NIST
 
Curso intensivo de soldadura electrónica en pdf
Curso intensivo de soldadura electrónica  en pdfCurso intensivo de soldadura electrónica  en pdf
Curso intensivo de soldadura electrónica en pdf
 
¿QUE SON LOS AGENTES FISICOS Y QUE CUIDADOS TENER.pptx
¿QUE SON LOS AGENTES FISICOS Y QUE CUIDADOS TENER.pptx¿QUE SON LOS AGENTES FISICOS Y QUE CUIDADOS TENER.pptx
¿QUE SON LOS AGENTES FISICOS Y QUE CUIDADOS TENER.pptx
 
COMPEDIOS ESTADISTICOS DE PERU EN EL 2023
COMPEDIOS ESTADISTICOS DE PERU EN EL 2023COMPEDIOS ESTADISTICOS DE PERU EN EL 2023
COMPEDIOS ESTADISTICOS DE PERU EN EL 2023
 
Calavera calculo de estructuras de cimentacion.pdf
Calavera calculo de estructuras de cimentacion.pdfCalavera calculo de estructuras de cimentacion.pdf
Calavera calculo de estructuras de cimentacion.pdf
 
Elaboración de la estructura del ADN y ARN en papel.pdf
Elaboración de la estructura del ADN y ARN en papel.pdfElaboración de la estructura del ADN y ARN en papel.pdf
Elaboración de la estructura del ADN y ARN en papel.pdf
 

SOLUCIONARIO 1 EVSUMATIVA 2 AAA 2020

  • 1. ASIGNATURA ABASTECIMIENTOS DE AGUA Y ALCANTARILLADO - CIC404 HOJA Nº 01/04 PRE-DIMENSIONAMIENTO DE LA TOMA Y LÍNEA DE CONDUCCIÓN 1ero. AÑO BASE DE OBSERVACIÓN ESTADÍSTISTICA-CATASTRAL: 2019 AÑO DE INICIO DE OPERACIONES: 2025 i) CUADRO 01.- DEMANDA CATASTRAL: Zona "A:GH" Zona "B:HI" Zona "C:IJ" Total Poblac.total año 2019 ( hab. ) 26,500 17,750 8,750 53,000 Dotación 2019 (Lt./hab./día) 375.00 248.00 178.00 (Referencia:Catastro) Consumo prom.2019 (Lt./seg.) 115.02 50.95 18.03 184.00 AA.HH. PERIODO DE DISEÑO : 20 AÑOS (Dato) AÑO FINAL DE OPERACIÓN: 2045 ii) Zona "A:GH" AÑO POBLACIÓN INC. Pob/year 2019 26,500 hab. 2024 27,438 hab. 188 100.00% 2029 28,151 hab. 143 -45 76.06% 2034 28,729 hab. 116 -27 81.12% Justificación inicial para construcción en 1 etapa 2039 29,216 hab. 97 -19 83.62% 2044 29,638 hab. 84 -13 86.60% -68.42% Población en etapa de saturación 2049 30,010 hab. 74 -10 88.10% 2054 30,344 hab. 67 -7 90.54% Pob 2025 27,581 hab. Pob 2030 28,267 hab. Pob 2035 28,826 hab. Pob 2040 29,300 hab. Pob 2045 29,712 hab. CRECIMIENTO TARDÍO CON ÍNDICE DECRECIENTE= CRECIMIENTO LOGARÍTMICO FACULTAD DE INGENIERIA CIVIL Y ARQUITECTURA ESCUELA ACÁDEMICA PROFESIONAL DE INGENIERÍA CIVIL CUESTIONARIO 17: Evaluación sumativa 02-AAA2020.2g1 (23/12/2020) - Peso 5 Ing. Civil Clifton Paucar y Montenegro - REG. CIP. 45773 - C. M Sc. Ing.Hidráulica - UNI Para pre-dimensionar la capacidad requeridad de la toma y línea de conducción iniciamos con el cálculo del caudal de diseño en función a los datos suministrados. Iniciamos el proceso de estimación elaborando el cuadro de datos y calculando el consumo promedio en función a la dotación promedio CATASTRAL O REGLAMENTARIA (según términos de referencia, rentabilidad y tipo de proyecto). Calculamos la población futura en funcIón al periodo de diseño y las ecuaciones generadas usando el Método de Incrementos cada 5 años en función a la población base de observación (2019) hasta el año final del período de diseño que según cálculo económico efectuado es de 20 años, partiendo del año de inicio de operación (2025) + 20 años = Año 2045 Ecuación promedio representativa de tres o mas ecuaciones que definen el crecimiento poblacional (Pto inical:-Pto Final): FINES ACADÉMICOS PARA DETERMINAR LA ECUACIÓN CARACTERÍSTICA DEL TIPO DE CRECIMIENTO EN EL QUE SE ENCUENTRA LA POBLACION TENEMOS QUE INTERPOLAR LOS DATOS CENSALES A PERIODOS CONSTANTES DE 5AÑOS Y ENCONTRAREMOS LA CURVA CON UN COEF. DE CORRELACIÓN LO MÁS CERCANO O IGUAL A 1, QUE ES LA QUE MEJOR 35,000 Xi i e Y 00033 . 0 22222  
  • 2. Zona "B:HI" AÑO POBLACIÓN INC. Pob/year 2019 17,750 hab. 2024 17,758 hab. 2 100.00% 2029 17,766 hab. 2 0 100.00% 2034 17,774 hab. 2 0 100.00% Justificación inicial para construcción en 1 etapa 2039 17,782 hab. 2 0 100.00% 2044 17,790 hab. 2 0 100.00% 0.00% Población en etapa de crecimiento 2049 17,798 hab. 2 0 100.00% 2054 17,806 hab. 2 0 100.00% Pob 2025 17,760 hab. Pob 2030 17,768 hab. Pob 2035 17,776 hab. Pob 2040 17,784 hab. Pob 2045 17,792 hab. CRECIMIENTO TARDÍO CON ÍNDICE DECRECIENTE= CRECIMIENTO LOGARÍTMICO AL TENER UN CRECIMIENTO ANUAL PORCENTUAL CONSTANTE DECRECIENTE PERCÁPITE POSITIVO ES APLICABLE EN ESTE CASO LA EXPRESIÓN DE CAPEN VALIDADA CON EVALUACIONES ADICIONALES SOCIO-ECONÓMICAS Y EXPANSIÓN FÍSICA PARA DETERMINAR LA ECUACIÓN CARACTERÍSTICA DEL TIPO DE CRECIMIENTO EN EL QUE SE ENCUENTRA LA POBLACION TENEMOS QUE INTERPOLAR LOS DATOS CENSALES A PERIODOS CONSTANTES DE 5AÑOS Y ENCONTRAREMOS LA CURVA CON UN COEF. DE CORRELACIÓN LO MÁS CERCANO O IGUAL A 1QUE ES LA QUE MEJOR REPRESENTA EN TEORIA EL CRECIMIENTO HISTÓRICO DE LA ZONA OBSERVANDO LA CURVA TIENE LA ÚLTIMA ETAPA DE CRECIMIENTO: TARDÍO (Saturación), CUYO TIPO DE CRECIMIENTO SE ENCUENTRA DIRECTAMENTE INFLUENCIADO POR ASPECTOS DE TIPO ESPACIAL, SOCIAL Y ECONÓMICO. Ecuación promedio representativa de tres o mas ecuaciones que definen el crecimiento poblacional (Pto inical:-Pto Final): FINES ACADÉMICOS PARA DETERMINAR LA ECUACIÓN CARACTERÍSTICA DEL TIPO DE CRECIMIENTO EN EL QUE SE ENCUENTRA LA POBLACION TENEMOS QUE INTERPOLAR LOS DATOS CENSALES A PERIODOS CONSTANTES DE 5AÑOS Y ENCONTRAREMOS LA CURVA CON UN COEF. DE CORRELACIÓN LO MÁS CERCANO O IGUAL A 1, QUE ES LA QUE MEJOR REPRESENTA EN TEORIA EL CRECIMIENTO HISTÓRICO DE LA ZONA OBSERVANDO LA CURVA TIENE LA ÚLTIMA ETAPA DE CRECIMIENTO: TARDÍO (Saturación), CUYO TIPO DE CRECIMIENTO SE ENCUENTRA DIRECTAMENTE INFLUENCIADO POR ASPECTOS DE TIPO ESPACIAL, SOCIAL Y ECONÓMICO. y = -46.048x2 + 946.76x + 25667 R² = 0.9985 0 5,000 10,000 15,000 20,000 25,000 30,000 35,000 2019 2024 2029 2034 2039 2044 2049 2054 POBLACIÓN INC. Pob/year Polinómica (POBLACIÓN) y = 17742e0.0004x R² = 1 8,000 10,000 12,000 14,000 16,000 18,000 20,000 Xi i e Y 00067 . 0 6677  
  • 3. Rpta. A) Zona B Zona "C:IJ" AÑO POBLACIÓN INC. Pob/year 2019 8,750 hab. 2024 14,569 hab. 1,164 100.00% 2029 25,003 hab. 2,087 923 179.30% 2034 45,206 hab. 4,041 1,954 193.63% Justificación inicial para construcción en 1 etapa 2039 89,771 hab. 8,913 4,872 220.56% 2044 213,474 hab. 24,741 15,828 277.58% 324.88% Población en etapa de crecimiento geométrico 2049 740,023 hab. 105,310 80,569 425.65% 2054 5,994,336 hab. 1,050,863 945,553 997.88% Justificación inicial para construcción en varias etapas Pob 2025 16,656 hab. Pob 2030 29,044 hab. Pob 2035 54,119 hab. Pob 2040 114,512 hab. Pob 2045 318,784 hab. CRECIMIENTO TEMPRANO CON ÍNDICE CRECIENTE= CRECIMIENTO GEOMÉTRICO POR LO TANTO LA CURVA TIENE UN CRECIMIENTO GEOMETRICO CON ECUACION EXPONENCIAL COMO SE MUESTRA EN EL GRAFICO. AL TENER UN CRECIMIENTO ANUAL PORCENTUAL, POSITIVO ES APLICABLE EN ESTE CASO ES EL QUE MEJORES RESULTADOS REPORTA EL USO DE LA EXPRESIÓN DE CAPEN OBSERVANDO LA CURVA TIENE LA ÚLTIMA ETAPA DE CRECIMIENTO: TARDÍO (Saturación), CUYO TIPO DE CRECIMIENTO SE ENCUENTRA DIRECTAMENTE INFLUENCIADO POR ASPECTOS DE TIPO ESPACIAL, SOCIAL Y ECONÓMICO. AL TENER UN CRECIMIENTO ANUAL PORCENTUAL CONSTANTE NO ES APLICABLE EN ESTE CASO LA EXPRESIÓN DE CAPEN DE ACUERDO A LAS CONSIDERACIONES INCIALES DE DISEÑO(PARÁMETROS DE CONTORNO). DEBERÁ USARSE OTRA ECUACIÓN BASADA EN ESTUDIOS SOCIO-ECONÓMICOS PARA DETERMINAR LA ECUACIÓN CARACTERÍSTICA DEL TIPO DE CRECIMIENTO EN EL QUE SE ENCUENTRA LA POBLACION TENEMOS QUE INTERPOLAR LOS DATOS CENSALES A PERIODOS CONSTANTES DE 5 AÑOS Y ENCONTRAREMOS LA CURVA CON UN COEF. DE CORRELACIÓN LO MÁS CERCANO O IGUAL A 1 QUE ES UNA DE LAS QUE MEJOR REPRESENTA EN TEORIA EL CRECIMIENTO HISTÓRICO DE LA ZONA Ecuación promedio representativa de tres o mas ecuaciones que definen el crecimiento poblacional (Pto inical:-Pto Final): FINES ACADÉMICOS 0 2,000 4,000 6,000 8,000 2019 2024 2029 2034 2039 2044 2049 2054 POBLACIÓN INC. Pob/year Exponencial (POBLACIÓN) y = 2570.5x6 - 60081x5 + 554984x4 - 3E+06x3 + 6E+06x2 - 7E+06x + 3E+06 R² = 0.9999 -1,000,000 0 1,000,000 2,000,000 3,000,000 4,000,000 5,000,000 6,000,000 7,000,000 2019 2024 2029 2034 2039 2044 2049 2054 i i X Y 00088 . 0 8 . 58  
  • 4. ETAPAS CONSTRUCTIVAS A CONSIDERAR (Verif. 1): 1 ETAPA iii) Se usará el cuadro de Relación: Diámetro versus Velocidad Económica ( Simón Arocha Ravelo - Normas INOS-Venezuela-1978) ø(Pulg) 3 4 6 8 10 12 14 V máx (m/s) 0.70 0.75 0.80 0.90 1.00 1.10 1.20 Q ( lit/seg.) 3.05 5.89 14.14 28.27 49.09 77.75 115.45 ø(Pulg) 16 18 20 24 30 V máx (m/s) 1.25 1.30 1.40 1.60 1.60 Q ( lit/seg.) 157.1 206.76 274.9 452.39 729.6 DEMANDA CATASTRAL: Zona "A:GH" Zona "B:HI" Zona "C:IJ" Total Φecon.i Dotac. Correg. Zona B c / 5% Pob.Total año 2019( Hab. ) 26,500 17,750 8,750 53,000 Dotación 2019(L/h/d) 375.00 248.00 178.00 248.00 248.00 Consumo prom.2019(L/s) 115.02 50.95 18.03 184.00 18" Pob.Total año 2025(Hab. ) 27,581 17,760 16,656 61,997 17,760 Dotación Correg. 2025(L/h/d) 376.88 260.40 192.92 248.01 modif. por 260.40 Consumo prom.2025(Lt./seg.) 120.31 53.53 37.19 211.03 18" Pob.Total año 2030(Hab. ) 28,267 17,768 29,044 75,079 17,768 Dotación Correg. 2030(L/h/d) 378.04 273.42 206.80 248.03 modif. por 273.42 Consumo prom.2030(Lt./seg.) 123.68 56.23 69.52 249.43 20" Pob.Total año 2035(Hab. ) 28,826 17,776 54,119 100,721 17,776 Dotación Correg. 2035(L/h/d) 378.97 287.09 223.53 248.04 modif. por 287.09 Consumo prom.2035(Lt./seg.) 126.44 59.07 140.01 325.52 24" Pob.Total año 2040(Hab. ) 29,300 17,784 54,119 101,203 17,784 Dotación Correg. 2040(L/h/d) 379.74 301.44 223.53 248.06 modif. por 301.44 Consumo prom.2040(Lt./seg.) 128.78 62.05 140.01 330.84 24" Pob.Total año 2045(Hab. ) 29,712 17,792 114,512 162,016 17,792 Dotación Correg. 2045(L/h/d) 380.40 316.51 245.49 248.07 modif. por 316.51 Consumo prom.2045(Lt./seg.) 130.82 65.18 325.37 521.37 30" % de Incremento (2045), con respecto al año de evaluación catastral (2019): 283.4% Rpta. B) CAUDAL DE DEMANDA REAJUSTADA LA DOTACIÓN: 521.40 Lt./seg. CUADRO 02.- POBLACIÓN Y CONSUMOS PROMEDIOS Porcentaje que no es significativo en el dimensionameinto de los componentes, hecho que es demostrable con la asignación de diámetros económicos para cada Caudal (Arocha: Relación Diámetros vrs. Velocidad Económica); por lo que no se aplicara la metodologìa consistente en la construcciòn del sistema por etapas(baterias), recomendada para casos de crecimiento explosivo durante el periodo de diseño que generan gastos iniciales altos al usar parámetros de diseño correspondientes al último año, que hacen enviable el proyecto por la rentabilidad inicial. Elaboramos el cuadro de Población y consumos Prom.usando la Expresión de CAPEN (G=k P 0.125 ), ACONDICIONADA A LOS PARÁMETROS DEL CRECIMIENTO POBLACIONAL DEL PROYECTO, para calcular la demanda corregida para el año 2045, considerando el inicio de operación 2025 Se determina diámetros INICIALES DE DISEÑO (usando el cuadro de relación: Diámetro-Velocidad Económica, propuesto por Simón Arocha Ravelo-Normas INOS-Venezuela -1978 ó Esp. Técnicas del fabricante AMANCO) a efectos de CONFIRMAR LA PROPUESTA DE EJECUCIÓN POR ETAPAS Se observa que el Qprom. Total para el año 2035 representa 2.834 del año de base observación (2019) -1,000,000 POBLACIÓN INC. Pob/year Polinómica (POBLACIÓN)
  • 5. Rpta. C) ETAPAS CONSTRUCTIVAS A CONSIDERAR (Verif.2final): 1 ETAPA Valores de k de la Expresión de CAPEN corregida de acuerdo a los datos poblacionales 2007 (*) G = k P 0.125 Donde : G = Consumo por habitante en Lit/hab./día P = Población en miles k= 204.40 ( Ref.:Para una zona especifica de Venezuela - D.Lauria) Zona "A:GH" 248.96 Zona "B:HI" 173.1 Zona "C:IJ" 135.73 POBLACIÓN EN ETAPA DE SATURACIÓN Zona "A:GH" DIÁMETRO DE DISEÑO AL INICIO DE OPERAC. DIÁMETRO FINAL DE DISEÑO 2025: 18" 2030-2035: 24" Datos para Graficos : Zona "A:GH" Año 2019 26,500 100.0% 375.00 100.0% 115.02 100.0% 2025 27,581 104.1% 376.88 100.5% 120.31 104.6% 2030 28,267 106.7% 378.04 100.8% 123.68 107.5% 2035 28,826 108.8% 378.97 101.1% 126.44 109.9% 2040 29,300 110.6% 379.74 101.3% 128.78 112.0% 2045 29,712 112.1% 380.40 101.4% 130.82 113.7% Datos para Graficos : Zona "B:HI" Año 2019 17,750 100.0% 248.00 100.0% 50.95 100.0% 2025 17,760 100.1% 260.40 105.0% 53.53 105.1% 2030 17,768 100.1% 273.42 110.3% 56.23 110.4% 2035 17,776 100.1% 287.09 115.8% 59.07 115.9% Diámetro de conducción en función a la demanda sin considerar características físicas del terreno: Captación-Línea de conducción-redes % Consum. Población (Hab) % Poblac. Dotación (L./h/d) Población (Hab) % Poblac. Dotación (L./h/d) % Dotación Consumo (L/s.) Por tener un incremento de habitantes por año con característica cercana al tipo intermedio(lineal) y tardío (logarítmica) % Dotación Consumo (L/s.) % Consum. 90.0% 95.0% 100.0% 105.0% 110.0% 115.0% 2019 2025 2030 2035 2040 2045 Título del gráfico % Poblac. % Dotación % Consum.
  • 6. 2040 17,784 100.2% 301.44 121.5% 62.05 121.8% 2045 17,792 100.2% 316.51 127.6% 65.18 127.9% Datos para Graficos : Zona "C:IJ" Año 2019 8,750 100.0% 178.00 100.0% 18.03 100.0% 2025 16,656 190.4% 192.92 108.4% 37.19 206.3% 2030 29,044 331.9% 206.80 116.2% 69.52 385.6% 2035 54,119 618.5% 223.53 125.6% 140.01 776.5% 2040 114,512 1308.7% 223.53 125.6% 140.01 776.5% 2045 318,784 3643.2% 245.49 137.9% 325.37 1804.6% iii) Se tiene el siguiente perfil de una línea de aducción por gravedad Cuadro 01: Puntos A B C D E F Población (Hab) % Poblac. Dimensionamos la línea de conducción calculando el GASTO DE DISEÑO y la CARGA DISPONIBLE Dotación (L./h/d) % Dotación Consumo (L/s.) % Consum. 0.0% 50.0% 100.0% 150.0% 2019 2025 2030 2035 2040 2045 Título del gráfico % Poblac. % Dotación % Consum. 0.0% 50.0% 100.0% 150.0% 2019 2025 2030 2035 2040 2045 Título del gráfico % Poblac. % Dotación % Consum.
  • 7. Cotas Mín. 1,912.00 1,788.00 1,855.00 1,600.00 1,900.00 1,550.00 D.Horiz.(Acum.) 0.00 300.00 750.00 1,250.00 2,000.00 2,500.00 Qpdb2019 = 184.00 l/s (Caudal promedio de demanda catastral base año de observación) ANÁLISIS DE POSIBLES ETAPAS CONSTRUCTIVAS: Año/Coefic. K1 K2 Qpd K1 Qpd Inc.K1Qpd K2 Qpd Inc. K2 Qpd K3Qpd Distor.CC+RV 2019 1.100 2.800 184.000 202.400 0.00000 515.200 0.00000 Usamos: 0.0 0.0 l/s 2020 1.100 2.800 188.510 207.361 4.96100 527.828 12.62800 Usamos: 0.0 0.0 l/s 2021 1.100 2.800 193.020 212.322 4.96100 540.456 12.62800 Usamos: 0.0 0.0 l/s 2022 1.100 2.800 197.530 217.283 4.96100 553.084 12.62800 Usamos: 0.0 0.0 l/s 2023 1.100 2.800 202.040 222.244 4.96100 565.712 12.62800 Usamos: 0.0 0.0 l/s 2024 1.100 2.800 206.550 227.205 4.96100 578.340 12.62800 Usamos: 0.0 0.0 l/s 2025 1.155 2.660 211.030 243.740 16.53465 561.340 -17.00020 Usamos: 0.0 0.0 l/s 2026 1.155 2.660 218.710 252.610 8.87040 581.769 20.42880 Usamos: 0.0 0.0 l/s 2027 1.155 2.660 226.390 261.480 8.87040 602.197 20.42880 Usamos: 0.0 0.0 l/s 2028 1.155 2.660 234.070 270.351 8.87040 622.626 20.42880 Usamos: 0.0 0.0 l/s 2029 1.155 2.660 241.750 279.221 8.87040 643.055 20.42880 Usamos: 0.0 0.0 l/s 2030 1.213 2.527 249.430 302.496 23.27498 630.310 -12.74539 Usamos: 0.0 0.0 l/s 2031 1.213 2.527 264.650 320.954 18.45805 668.771 38.46094 Usamos: 0.0 0.0 l/s 2032 1.213 2.527 279.870 339.412 18.45806 707.231 38.46094 Usamos: 0.0 0.0 l/s 2033 1.213 2.527 295.090 357.870 18.45805 745.692 38.46094 Usamos: 0.0 0.0 l/s 2034 1.213 2.527 310.310 376.328 18.45806 784.153 38.46094 Usamos: 0.0 0.0 l/s 2035 1.273 2.401 325.520 414.513 38.18465 781.460 -2.69378 Usamos: 0.0 0.0 l/s 2036 1.273 2.401 326.580 415.863 1.34979 784.004 2.54469 Usamos: 0.0 0.0 l/s 2037 1.273 2.401 327.640 417.213 1.34979 786.549 2.54469 Usamos: 0.0 0.0 l/s 2038 1.273 2.401 328.700 418.562 1.34979 789.094 2.54469 Usamos: 0.0 0.0 l/s 2039 1.273 2.401 329.760 419.912 1.34979 791.638 2.54469 Usamos: 0.0 0.0 l/s 2040 1.337 2.281 330.840 442.352 22.43963 754.519 -37.11885 Usamos: 0.0 0.0 l/s 2041 1.337 2.281 368.950 493.307 50.95524 841.434 86.91433 Usamos: 0.0 0.0 l/s 2042 1.337 2.281 407.060 544.262 50.95524 928.348 86.91433 Usamos: 0.0 0.0 l/s 2043 1.337 2.281 445.170 595.218 50.95524 1015.262 86.91433 Usamos: 0.0 0.0 l/s 2044 1.337 2.281 483.280 646.173 50.95524 1102.177 86.91433 Usamos: 0.0 0.0 l/s 2045 1.404 2.167 521.370 731.956 85.78356 1129.593 27.41644 Usamos: 732.0 1,129.6 l/s Los valores establecidos en los estudios de validación y calibración de los parámetros de diseño de los coeficientes diarios y horarios son: Se solicita predimensionar la línea de conducción con la combinación de diámetros de mayor eficiencia según las características de los parámetros de demanda, tipo de material de las 800.000 1000.000 1200.000 Caudales de Diseño (Qp1, Qp2) 0.00000 20.00000 40.00000 60.00000 80.00000 100.00000 Inc.K1Qpd
  • 8. Q1diseño = 732.0 l/s CARGAS DISPONIBLES: Todo el tramo hasta donde empiezan las viviendas rurales AH : 1912 - 1550 = 362.00 m. Punto crítico E, tramo AE : 1912 - 1900 12.00 m. Cota Máx. 1,912.00 (Con fines Académicos) Puntos A B C D E F Cotas Mín. 1,912.00 1,788.00 1,855.00 1,600.00 1,900.00 1,550.00 D.H.(Parcial) 0.00 300.00 450.00 500.00 750.00 500.00 D.H.(Acum.) 0.00 300.00 750.00 1,250.00 2,000.00 2,500.00 D.I.(Parcial) 0.00 324.62 454.96 561.27 807.77 610.33 D.I.(Acumul.) 0.00 324.62 779.58 1,340.85 2,148.62 2,758.95 DESNIVELES: 124.00 57.00 312.00 12.00 362.00 Fin. PD Inicio de Operación Qdis.(2045)= 732 l/s (PD=20) 2045 2025 Cota nivel max. = 1912.00 m Cota nivel min. = 1912.00 m A : Fuente de abastecimiento B : Primer punto de cambio de pendiente de la tubería C : Segundo punto de cambio de pendiente de la tubería D : Tercer punto de cambio de pendiente de la tubería E : Cuarto punto de cambio de pendiente de la tubería (CRÍTICO) F : Quinto BM final de línea de conducción (Planta de Tratamiento - Altura mínima de llegada : 5.00 m. VALIDAR Pto. Crítico A B C D E F ( Se considera que la Planta de tratamiento y el Reservorio se ubicaran en la zona adyacente al punto G y en cotas superiores al existir viviendas rurales a partir de este punto) CÁLCULO DE TUBERÍAS -200.000 0.000 200.000 400.000 600.000 2019 2020 2021 2022 2023 2024 2025 2026 2027 2028 2029 2030 2031 2032 2033 2034 2035 2036 2037 2038 2039 2040 2041 2042 2043 2044 2045 K1 Qpd K2 Qpd Inc.K1Qpd Inc. K2 Qpd -50.00000 0.00000 50.00000 100.00000 Inc. K2 Qpd
  • 9. D. HORIZ (Acum.) 0.00 300.00 750.00 1,250.00 2,000.00 2,500.00 D. INCLIN (Acum.) 0.00 324.62 779.58 1,340.85 2,148.62 2,758.95 COTA 1,912.00 1,788.00 1,855.00 1,600.00 1,900.00 1,550.00 12.00 362.00 DIÁMETRO DE TUBERÍA Cota mínima de carga de agua sobre la tubería en la fuente de abastecimiento 1,912.00 m. ΔH : Diferencia de elevación entre A y D α : Coeficiente del diámetro L : Longitud total Q : Caudal de diseño ΔH = 362.00 m Cota de nivel mínimo de fuente de abastecimiento - Cota de reservorio LAF = 2,758.95 m α = 6.59E-07 Coeficiente del diámetro calculado POSIBLE COMBINACIÓN DE DIAMETROS α : Coeficiente del diámetro C : Coeficiente de rugosidad del material de la tubería D : Diametro de la tubería Buscando un coeficiente menor y un coeficiente mayor y próximos al coeficiente calculado C = 150 DIAMETRO (Pulg.) α 0.50 4.7400E+00 0.75 6.5798E-01 1.00 1.6209E-01 1.25 5.4677E-02 2.00 5.5430E-03 2.50 1.8698E-03 3.00 7.6945E-04 4.00 1.8955E-04 6.00 2.6313E-05 8.00 6.4820E-06 α 2 : Coef. del diametro menor 10.00 2.1866E-06 (Final) 12.00 8.9980E-07 = α 2 (comercial) 13.1310 5.8030E-07 = α calc. (teórico) C= 150 14.00 4.2473E-07 = α 1 (comercial) 16.00 2.2166E-07 20.00 7.4773E-08 24.00 3.0770E-08 = α 2 (comercial) 24.4641 2.8030E-08 = α calc. (teórico) C= 150 Para tubería GRPR(Poliester reforzado con fibra de vidrio rectificado) C =140 85 . 1 LQ H    87 . 4 85 . 1 3 10 72 . 1 D C    
  • 10. 36.00 4.2714E-09 = α 1 (comercial) 48.00 1.0522E-09 α 1 : Coef. del diametro mayor 60.00 3.5495E-10 (Final) 75.00 1.1973E-10 90.00 4.9272E-11 120.00 1.2138E-11 150.00 4.0944E-12 Los posibles diámetros a combinar en el tramo AF es de 14" y 12" de forma preliminar Luego de la verificación de conducción en elpunto crítico E, se tiene, que: Los diámetros a combinar son de 36" y 24" de forma preliminar PREDIMENSIONAMIENTO Y ANÁLISIS DEL TRAMO TOTAL AF: VERIFICACIÓN DE LA APLICACIÓN DE LA FÓRMULA DE HAZEN Y WILLIAM: Se debe cumplir: Tipo de flujo : Turbulento Re > 2300 > 106 Velocidad < 3 m/seg Diámetros mayores a 2" 104 - 106 Tuberías GRP: Poliester reforzado con fibra de vidrio (GRP) PAVCO ; C=150 > 2 x 105 ACERO: C=120 Si el flujo es de tipo Laminar entonces usar ecuación de DARCY Hallando la velocidad media: PVC : Alegret Breña & Martinez Valdés, 2019: 0.05 m (2" )< D < 1.85 m (74") 3.05 m/s > V V : Velocidad media del flujo 5° < T° < 25° Q : Caudal de diseño A : Area de la sección tubería A = 0.65669289 m2 36.00 pulgadas Antes Re>2,300 A = 0.29186351 m2 24.00 pulg. Antes Re>2,300 V = 1.11 m/seg OK usar H y W si 10,0000<Re<1,000,000 V = 2.51 m/seg OK usar H y W si 10,0000<Re<1,000,000 Tipo de flujo : Re : Velocidad media del flujo V : Caudal de diseño D : Area de la sección tubería υ : viscocidad del flujo 36.00 pulgadas Re = 112,776 OK usar H y W Es correcto usar Hazen William Tuberías CCP: Cilindro de acero revestido con concreto o mortero de cemento, reforzados helicoidalemnte con varilla de acero al carbón . C=120  D V   Re A Q V 
  • 11. DETERMINAMOS LAS LONGITUDES ÓPTIMAS COMBINANDO DIÁMETROS ΔH = 362.00 m Cota de nivel mínimo de carga de agua en fuente de abastecimiento - Cota de reservorio Qdis.(2045)= 732 Lts/seg (PD=20) LAF= 2,758.95 m α 1 : 4.2473E-07 14.0 pulg. Coef. del diametro mayor (Inicial) α 2 : 7.6945E-04 12.0 pulg. Coef. del diametro menor (final) L'AF= 1.05 LAF= 2896.8975 m J = 245.13 m OK 116.87 X : Distancia correspondiente al diametro mayor ΔH : Diferencia de elevación entre A y E L' : Longitud afectada en un 5% ΔH = 362.00 m Cota de nivel mínimo de carga de agua en fuente de abastecimiento - Cota de reservorio ΔHpt= 1.00 m Presión mínima en cualquier punto de la tubería por conducción X = 1662 m Long. De tubería mayor : 14.00 Pulg. :α 1 4.2473E-07 L-X = 1,097.00 m Long. De tubería menor : 12.00 Pulg. :α 2 8.9980E-07 2,759.00 OK, ES PROCEDENTE LA COMBINACIÓN DE DIÁMETROS CHEQUEANDO PARA LA LONGITUD CRÍTICA AE ΔH : Diferencia de elevación entre A y D α : Coeficiente del diámetro L : Longitud total Q : Caudal de diseño CARGA DISP. ΔHAE = 12.00 m Cota de nivel mínimo de carga de agua en fuente de abastecimiento - Cota en E LAE= 2,148.62 m α = 2.8033E-08 Coeficiente del diámetro calculado 2.8033E-08 Requisito : J < ΔH CONSIDERANDO : 36.00 Pulg. :α 1 4.2714E-09 J = 1.83 12.00 Requisito : J < ΔH OK 10.17 COMBINAR DIÁMETROS PARA OPTIMIZAR EL DISEÑO L'AE= 1.05 LAE= 2256.05 m ΔH = 12.00 m Cota de nivel mínimo de carga de agua en fuente de abastecimiento - Cota en E RESP. 1: TRAMO INIC. 36.00 pulg. ΔHpt= 1.00 m Presión mínima en cualquier punto de la tubería por conducción RESP. 2: TR.AE DE 24" = 346.00 m CARGA DISPONIBLE ΔHAE = EVALUAMOS LA DIFERENCIA ΔHAE - J   2 1 85 . 1 85 . 1 2 '          Q Q L H X 85 . 1 1 Q L J AE   85 . 1 LQ H    85 . 1 1 Q L J AF  
  • 12. X = 346 m Long. De tubería mayor : 36.00 pulg. :α 1 4.2714E-09 Rpta. D) 36.00 pulg. L-X = 1,802.60 m Long. De tubería menor : 24.00 pulg. :α 2 3.0770E-08 2,148.60 Para las redes 14.00 pulg. :α 2 4.2473E-07 RESP. 3: TRAMO FINAL 24.00 pulg. OK, ES PROCEDENTE LA COMBINACIÓN DE DIÁMETROS RESP. 4: TR.AE DE 24" = 1,802.60 m CHEQUEO TRAMO CRÍTICO AE CON LA COMBINACIÓN DE DIÁMETROS: TOTAL TRAMO: AE 2,148.60 m. TOTAL TRAMO: EF 610.33 m. TOTAL TRAMO: AF 2,758.93 m. J = 11.34 12.00 OK 0.66 AUMENTAR LONGITUD DE DIÁMETRO MAYOR PARA OPTIMIZAR EL DISEÑO ALTURA MÍNIMA DE CARGA EN CADA NUDO 0.5 mca CHEQUEANDO POR TRAMOS: Cota mínima de carga de agua sobre la tubería en la fuente de abastecimiento 1,912.00 m.c.a.: metros de columna de agua TRAMO AB Cota Tub. B= 1,788.00 ΔHAB = 124.00 Cota de nivel mínimo de carga de agua en fuente de abastecimiento - Cota en B LAB = 324.62 36.00 Pulg. L'AB=1.05* LAB = 340.85 J = 0.29 OK Requisito : J < ΔH Cota Piez B= 1911.71 Cota Tub. B= 1788.00 m.c.a.= 123.71 Altura mín. 0.50 OK TRAMO AC Cota Tub. C= 1,855.00 ΔHAC = 57.00 Cota de nivel mínimo de carga de agua en fuente de abastecimiento - Cota en C LAC = 779.58 36.00 Pulg. L'AC=1.05* LAC = 818.56 J = 0.70 OK Requisito : J < ΔH Cota Piez C= 1911.30 Cota Tub. C= 1855.00 m.c.a.= 56.30 Altura mín. 0.50 OK TRAMO AD Cota Tub. D= 1,600.00 ΔHAD = 312.00 Cota de nivel mínimo de carga de agua en fuente de abastecimiento - Cota en D LAD = 1,340.85 36.00 Pulg. CARGA DISPONIBLE ΔHAE = EVALUAMOS LA DIFERENCIA ΔHAE - J 85 . 1 1 Q L J AC   85 . 1 1 Q L J AB       85 . 1 2 1 Q L L L J X AE X      85 . 1 1 Q L J AD  
  • 13. L'AD=1.05* LAD = 1,407.89 J = 1.20 OK Requisito : J < ΔH Cota Piez D= 1910.80 Cota Tub. D= 1600.00 m.c.a.= 310.80 Altura mín. 0.50 OK TRAMO AE Cota Tub. E= 1,900.00 ΔHAE = 12.00 Cota de nivel mínimo de carga de agua en fuente de abastecimiento - Cota en D LAE = 2,148.62 36.00 Pulg. + 24.00 Pulg. COMBINACIÓN DE DIÁMETROS L'AE=1.05* LAE = 2,256.03 36.00 Pulg. 346.00 m J = 11.34 OK Requisito : J < ΔH 24.00 Pulg. 1802.60 m Cota Piez E= 1900.66 2148.60 m Cota Tub. E= 1900.00 m.c.a.= 0.66 Altura mín. 0.50 OK TRAMO AF Cota Tub. F= 1,550.00 ΔHAF = 362.00 Cota de nivel mínimo de carga de agua en fuente de abastecimiento - Cota en F LAF = 2,758.95 36.00 Pulg. + 24.00 Pulg. COMBINACIÓN DE DIÁMETROS L'AF=1.05* LAF = 2,896.90 36.00 Pulg. 346.00 m J = 15.09 OK Requisito : J < ΔH 24.00 Pulg. 2412.95 m Cota Piez F= 1896.91 2758.95 m Cota Tub. F= 1550.00 m.c.a.= 346.91 Altura mín. 0.50 OK CONSIDERANDO UN SOLO DIAMETRO PARA TODA LA LONGITUD (AF) Cota Tub. F= 1,550.00 ΔHAF = 362.00 LAF = 2,758.95 L'AF=1.05* LAF = 2,896.90 36.00 Pulg. J = 2.47 Cota Piez F= 1909.53 Cota Tub. F= 1550.00 Queda demostrado que un 5% de aumento de longitud equivale a 1 metro 85 . 1 1 Q L J AE       85 . 1 2 1 Q L L L J X AE X          85 . 1 2 1 Q L L L J X AE X     
  • 14. m.c.a.= 359.53 - 346.91 = 12.62 mca AHORRO Altura mín. 0.50 OK m.c.a.: metros de columna de agua Qd = 732.00 l/s L(acum.) = 2,758.95 m USAR TUBERÍA 36 pulgadas 36" 346.00 m. POLIETILENO DE ALTA DENSIDAD (HDPE) POLIVINILO DE CLORURO (PVC) PESADO RESP. 1: TRAMO FINAL 24 pulgadas 24" 2,413.00 m. POLIVINILO DE CLORURO (PVC) PESADO TOTAL LONGITUD AE AF 2,759.00 m CLASE DE TUBERÍA Cota Inicio(máx.) 362.00 m. USAR TUBERIA PVC CLASE 5, 7.5, 10, 15 DE 8¨ y 6 ¨ s/esquema SE ANALIZA EL TRAMO HASTA EL PUNTO CRÍTICO AE: En función de la carga disponible entre A y E α AE = En función a los diámetros para tubería PVC α X" = 0 C = 150 α Y" = 0 α Z" = 0 En función a los diámetros para tubería ASBESTO α N" = 0 CEMENTO C = 120 α M" = 0 α R"= 0 En función a los diámetros para F tubería F G α S" = 0 CARGA DISPONIBLE EN PARA DETERMINAR LA CLASE DE TUBERÌA A USAR SE INICIARA CON CLASE C-5, CONSIDERAN LA COTA MAXIMA DEL NIVEL DE AGUA, COMO EL PUNTO D TIENE DIFERENCIA DE COTA CRÍTICA (DIFERENCIA DE COTAS MAYOR CON LA CARGA HIDRÁULICA), DE SPBREPASA VALORES DE 250 M (C-25) SE CONSIDERARA LA CONSTRUCCIÓN DE UN PUENTE USANDO TUBERÍA CLASE 25
  • 15. C = 110 α T" = 0 α R"= 0 J AE = 15.09 ( Usando ΦHDPE 8" Y 6" ) m.c.a F = 346.91 Requerim. M.c.a F(Pta.Tr) = 5.00 OK VALIDAR J EF = XXXXXX ( Usando Φfºgº AE1' ,Φ pvcE1E2" y Φ pvc E2F" ) Si ΔEEF = 350.00 m SE NECESITA CÁMARAS O VÁLVULAS ROMPEPRESIÓN iv) POR DATO SABEMOS QUE EN EL TRAMO ABCDE SE USARA TUBERÍA DE HDPE, POR LO QUE CHEQUEAMOS LA PERDIDA DE CARGA JAC. CON LOS α CORRESPONDIENTES A LA COMBINACIÓN DE DIÁMETROS DE A hacia F USANDO SÓL TUBERÍA HDPE-PVC-ITINTEC-DIÁMETRO INTERNO EN PULGADAS. NECESITAMOS PRESIÓN SOLO PARA PASAR EL PUNTO E LUEGO DE ENCONTAR UNA DIFERENCIA MAYOR A 50 METROS DE ALTURA DE (Topografía Plana de Poblaciones A, B y C, sólo con fines académicos) Con éstos datos elaboramos el perfil longitudinal de la línea de Conducción 240 250 260 270 280 290 300 310 320 330 340 350 Cotas Mín. Cotas PLANTA DE TRATAMIENTO PLANO ESTÁTICO DE PRESIONES 360 m = PLANO ESTÁTICO DE PRESIÓN LIMITANTE (100 Lb./Pulg2 <>70 m. de agua) PLANO ESTÁTICO LIMITE DE PRESIONES PARA DETERMINAR LA CLASE DE TUBERÍA: 50 M. DE ALTURA DE AGUA DESDE EL PUNTO A. PLANO ESTÁTICO LIMITE DE PRESIONES PARA DETERMINAR LA CLASE DE TUBERÍA: 75 M. DE ALTURA DE AGUA DESDE EL PUNTO A. Pob. A Pob. B Pob. C
  • 16. Cota Máx. 1,912.00 Puntos A B C D E F G H I J Cotas Mín. 1,912.00 1,788.00 1,855.00 1,600.00 1,900.00 1,550.00 1,550.00 1,550.00 1,550.00 1,550.00 D.H.(Parcial) 0.00 300.00 450.00 500.00 750.00 500.00 1,000.00 1,000.00 1,000.00 1,000.00 D.H.(Acum.) 0.00 300.00 750.00 1,250.00 2,000.00 2,500.00 3,500.00 4,500.00 5,500.00 6,500.00 D.I.(Parcial) 0.00 324.62 454.96 561.27 807.77 610.33 1,000.00 1,000.00 1,000.00 1,000.00 (Verific.Datos) D.I.(Acumul.) 0.00 324.62 779.58 1,340.85 2,148.62 2,758.95 3,758.95 4,758.95 5,758.95 6,758.95 Φ 36" 36"- 24" 24" 24" 24" 24" 14" MATERIAL Φ PVC PVC PVC J 0.00 0.29 0.70 1.20 11.34 15.09 Cota Piezom. 1,912.00 1,911.71 1,911.30 1,910.80 1,900.66 1,896.91 1,896.91 1,896.91 1,896.91 1,896.91 VERIFICACIÓN 0.00 123.71 56.30 310.80 0.66 346.91 346.91 346.91 346.91 346.91 OK OK OK OK OK CLASE Φ 5 5,7.5,10,15,20,25 5,7.5,10,15,20,25 5,7.5,10,15,20,25 5,7.5,10,15,20,25 5,7.5,10,15,20,25 5,7.5 5,7.5 5,7.5 5,7.5 D.V.(parcial) 124.00 0.00 67.00 -188.00 112.00 -350.00 -350.00 -350.00 -350.00 COTA PIEZOMÉTRICA F: 1,896.91 m COORDENADAS DE UBICACIÓN DE EMPALMES DE DIÁMETROS DE TUBERÍAS, OBRAS DE ARTE, PUENTES Y CAMARAS O VÁLVULAS ROMPE-PRESIÓN 1ER. PRE-DIMENSIONAMIENTO CON CARGAS DINÁMICAS EN EXTREMOS (CRÍTICAS) OBRAS DE ARTE COTA Intersecciones de Clases de Tuberías C= 150 PUENTE 1 Altura de agua (mca) para C-5, Tramo CD: 50 mca CAMARA ROMPE PRESION 1 Cota inicial para usar C-5 +1912 1,912.00 m CAMARA ROMPE PRESION 2 Cota final para usar C-5 +1912-1.2-50= 1,860.80 m CAMARA ROMPE PRESION 3 Por semejanza de triángulos CAMARA ROMPE PRESION 4 HDPE (Inst. Exterior) 150 160 170 180 190 200 210 220 230 240 A B C D E F G H I J Cotas Mín. PLANTA DE TRATAMIENTO Y RESERVORIO PUENTE (NO) CAJAS ROMPEPRESIÓN (NO) CISTERNA, CASETA DE BOMBEO Y RESERVORIO PLANO ESTÁTICO LIMITE DE PRESIONES PARA DETERMINAR LA CLASE DE TUBERÍA: 100 M. DE ALTURA DE AGUA DESDE EL PUNTO A. PLANO ESTÁTICO LIMITE DE PRESIONES PARA DETERMINAR LA CLASE DE TUBERÍA: 150 M. DE ALTURA DE AGUA DESDE EL PUNTO A.
  • 17. CAMARA ROMPE PRESION 5 Puntos C D DH.acum. 750.00 1,250.00 CAMARA ROMPE PRESION 6 Cotas 1,855.00 1,600.00 CAMARA ROMPE PRESION 7 DIacum. 779.58 1,340.85 x'CD CAMARA ROMPE PRESION 8 X'CD -11.37 m CAMARA ROMPE PRESION 9 C1 792.34 DIparc.C-C1= 12.8 m CAMARA ROMPE PRESION 10 1,860.80 DIacum.A-C1= 792.34 m OK CAMARA ROMPE PRESION 11 CAMARA ROMPE PRESION 12 Distancia inclinado acum. A-C1 Clase C5= 816.00 m. Tubería C5 de diámetro: 36" 346.00 m. Tubería C5 de diámetro: 24" 470.00 m. Altura de agua (mca) para C7.5, Tramo CD: 75 mca CAMARA ROMPE PRESION 13 Cota inicial para usar C-7.5 +1912 1,912.00 m CAMARA ROMPE PRESION 14 Cota final para usar C-7.5 +1912-1.2-75= 1,835.80 m CAMARA ROMPE PRESION 15 Por semejanza de triángulos CAMARA ROMPE PRESION 16 CAMARA ROMPE PRESION 17 Puntos C D CAMARA ROMPE PRESION 18 DH.acum. 750.00 1,250.00 CAMARA ROMPE PRESION 19 Cotas 1,855.00 1,600.00 CAMARA ROMPE PRESION 20 DIacum. 779.58 1,340.85 CAMARA ROMPE PRESION 21 x''CD RESERVORIO X''CD 37.65 m C2 821.84 DIparc.C-C2= 42.3 m REQUERIMIENTO FINAL: 1,835.80 DIacum.A-C2= 821.84 m OK Norma de Fabricación ASTM ó ITINTEC Tubería C5-36" incluído 3% desperd.= Unid. 163.2 tubos Distancia inclinado acum.A-C2 C5-C7.5= 821.84 m. Tubería C5 de diámetro: 36" 346.00 m. Tubería C5 de diámetro: 24" 475.84 m. Tubería C5 de diámetro: 24" 470.00 m. Tubería C7.5 de diámetro: 24" 5.84 m. Altura de agua (mca) para C10, Tramo CD: 100 mca Cota inicial para usar C-7.5 +1912 1,912.00 m Cota final para usar C-7.5 +1912-1.2-100= 1,810.80 m Por semejanza de triángulos Puntos C D DH.acum. 750.00 1,250.00 Cotas 1,855.00 1,600.00 DIacum. 779.58 1,340.85
  • 18. x'''CD X'''CD 86.67 m C3 876.87 DIparc.C-C3= 97.3 m 1,810.80 DIacum.A-C3= 876.87 m OK DHparc.x-D= 413.33 m DIacum.C3-D= 463.98 m Para verificación final Distancia inclinado acum.A-C3 Clase C10= 876.87 m. Tubería C5 de diámetro 36" 346.00 m. Tubería C5 de diámetro 24" 530.87 m. Tubería C5 de diámetro: 24" 470.00 m. Tubería C7.5 de diámetro: 24" 5.84 m. Tubería C10 de diámetro: 24" 55.03 m. Rpta. E) 1810.80 msnm Altura de agua (mca) para C10, Tramo DE: 100 mca Ubicación del Puente: P1 Cota inicial para usar C-7.5 +1912 1,912.00 m Punto. C3-Cota inicial C10-4 ": 1,810.80 Cota final para usar C-7.5 +1912-1.2-100= 1,810.80 m Punto. D3-Cota final C10-4 ": 1,810.80 (Con fines académicos) Por semejanza de triángulos Longitud efectiva tramo C3-D3: 940.33 m Puntos D E DH.acum. 1,250.00 2,000.00 Cotas 1,600.00 1,900.00 DIacum. 1,340.85 2,148.62 x'DE X'DE 223.00 m 306.86 D3 DIparc.E-D3= 306.86 m 1,810.80 DIacum.E-D3= 306.86 m DHparc.x-D3= 527.00 m DIacum.A-D3= 1817.20 m OK DIacum.D-D3= 500.91 m Para verificación final VERIFICACIÓN DE DIÁMETROS Y CLASE DE TUBERÍAS HASTA EL PUENTE P1: (BM1) Distancia inclinada acum. A-D3: 1817.20 m Tubería C5 de diámetro 36" 346.00 m. Clase C5 Tubería C5 de diámetro 24" 470.00 m. Clase C5 Tubería C7.5 de diámetro 24" 60.87 m. Clase C7.5 Tubería C10 de diámetro 24" 940.33 m. Clase C10 Puente 01 1,817.20 m. OK Altura de agua (mca) para C7.5, Tramo DE: 75 mca Cota inicial para usar C-7.5 +1912 +1912 m Cota final para usar C-7.5 +1912-11.34-75= 1,825.66 m Por semejanza de triángulos
  • 19. Puntos D E DH.acum. 1,250.00 2,000.00 Cotas 1,600.00 1,900.00 DIacum. 1,340.85 2,148.62 x''ED X''DE 185.85 m 292.34 D2 C5: DIparc.E-D2= 292.34 m 1,825.66 DIacum.E-D2= 292.34 m DHparc.x-D2= 564.15 m DIacum.A-D2= 1854.35 m OK Distancia inclinada acum. A-D2: 1854.35 m Tubería C5 de diámetro 36" 346.00 m. Clase C5 Tubería C5 de diámetro 24" 470.00 m. Clase C5 Tubería C7.5 de diámetro 24" 60.87 m. Clase C7.5 Tubería C10 de diámetro 24" 940.33 m. Clase C10 Puente 01 Tubería C7.5 de diámetro 24" 37.15 m. Clase C7.5 Tubería C5 de diámetro 24" 292.34 m. Clase C5 DIparc.E-D2= VERIFICACIÓN Y CONTROL SECUENCIAL (BM2): Distancia inclinada acum. A-E: 2146.69 m +TR:C3-D3: 964.89 = 3111.58 - 940.33 = 2,171.3 m COTA EN EL PTO. E. = 2,148.62 m ERROR ACUMULADO POR USAR PRESIONES DINÁMICA EN LOS PTOS. EXTREMOS A CORREGIR SI GENERARN DISTORSIONES SIGNIFICATIVAS(TÉCNICO-ECONÓMICO) 22.63 m COORDENADAS DE UBICACIÓN DE CÁMARAS O VÁLVULAS ROMPE-PRESIÓN Ubicación de la 1CR Se usara la Formula de Manning D = 1.548(nQ/s^1/2)^3/8 Cota dinámica de llegada al Pto. E. 1,900.66 m Diam.(pulg.) D(m), Q(m3/seg.), S(m/m) Cota topográfica del pto. E 1,900.00 m 0 0.00 n(pvc) = 0.009 mca disponible en el pto.E. 0.66 m. ¡RECALCULAR¡ 0.00 0.00 #¡DIV/0! m. 0.00 0.00 #¡DIV/0! pulgadas Cota dinámica de llegada al Pto. F. 1,896.91 m Se usara : 8" Cota topográfica del pto. E 1,550.00 m COTA USANDO 8" 3,679.0 msnm. mca disponible en el pto.E. 346.91 m. DI 1CR 78.6 m. mca mín. requerido en E (Reserv. Transv.) 5.00 m OK DH 1CR 35.2 m. mca mín. requerido en E (Reserv. Distrib.) 75.00 m OK CONCLUSIONES INICIALES: DATOS TOPOGRÁFICOS DE LAS ZONAS A (tramo FG), B(tramo GH) Y C(tramo HI): Tramo longitudinal FG: 1,000.00 m Cota G : 1,550.00 Por la escesiva carga hidráulica generada en el tramo EF SE CONSIDERARAN LAS CONSTRUCCIÓN DE CAMARAS ROMPEPRESIÓN O ADQUISICIÓN DE VÁLVULAS REDUCTORAS Se plantean DOS ESCENARIOS CONSTRUCTIVOS, 1er. Escenario) La construcción de un sólo rservorio principal en el Pto E para abastecer a las tres poblaciones A,B, y C, del tipo semi- enterrado por la capacidas de carga hidráulica de llegada, reduciendo las presiónes con válvulas en la distribución de la red de agua para no sorepasar los 50 mca exigidos en el reglamento. 2do. Escenario), del Reservorio Principal en E se distribuirá a reservorios zonales en la población A(tramo GH), población B (tramo HI) y población C (tramo IJ), para redistribuir el agua por gravedad.
  • 20. Tramo longitudinal GH: 1,000.00 m Cota H : 1,550.00 Tramo longitudinal HI: 1,000.00 m Cota I : 1,550.00 Pto A: 1,912.00 24" ZONA "A" 14" ZONA "B" 14" ZONA "C" 14" Puntos E Cota ResP F G H I DH.acum. 2,000.00 2,500.00 3,500.00 4,500.00 5,500.00 Cotas 1,900.00 1868.94 1,550.00 1,550.00 1,550.00 1,550.00 DIacum. 2,148.62 1784.33 2,758.95 3,758.95 4,758.95 5,758.95 J 11.34 1699.71 15.09 99.71 184.33 268.94 Cota Piezom. 1,900.66 1,896.91 1812.29 1727.67 1643.06 Manteniendo el NAM=Cterr.E Carga Hid. (mca) 0.66 346.91 262.29 177.67 93.06 OK Art. 4.5 OS.050-RNE 200.29 115.67 31.06 MÁXIMA PRESIÓN SEGÚN REGLAMENTO (mca) 62.00 62.00 62.00 NUEVA CARGA PIEZOMÉTRICA PARA USAR UN SOLO RESERVORIO 50.00 50.00 50.00 Rpta. F) 1868.90 msnm E C3 0.00 1,550.00 ZONA "A" ZONA "B" ZONA "C" F G H I SE PROCEDE A EFECTUAR LOS CÁLCULOS CONSIDERANDO UN RESERVORIO PRINCIPAL, PARA ABASTECER A LA POBLACIÓN, A, B Y C, TIPO ENTERRADO UBICADO EN EL PUNTO : E ESCENARIO DE SISTEMA DE DISTRIBUCIÓN DE AGUA 1: DISTRIBUCIÓN POR GRAVEDAD DE 1 SOLO RESERVORIO PRINCIPAL AÑO BASE DE OBSERVACIÓN ESTADISTISTICA-CATASTRAL 2019 Año inicio de Operac. 2025 Año final del Pdiseño 2045 i) POBLACIÓN TOTAL 2045 100.0% 162,016 habitantes 35,307 Viv. Viviendas A : Densidad poblacional promedio: 8 Hab. / Viv. 18.3% 29,712 N° Viviendas 3,714 Viviendas B : Densidad poblacional promedio: 6 Hab. / Viv. 11.0% 17,792 N° Viviendas 2,965 Viviendas C : Densidad poblacional promedio: 4 Hab. / Viv. 70.7% 114,512 N° Viviendas 28,628 162,016 CUADRO 01a.- SE PUEDE CONSIDERAR UN SOLO RESERVORIO PRINCIPAL Se deberá verificar la dotación promedio CATASTRAL, en función al consumo promedio cuantificado por tipo de EDIFICACIÓN Y HABITANTE, así como el consumo promedio en litros/seg, para el año final del periodo de diseño establecido. PLANTA DE TRATAMIENTO Y RESERVORIO PRINCIPAL ENTERRADO CISTERNA + RESERVORIO ZONAL A CISTERNA + RESERVORIO ZONAL B CISTERNA + RESERVORIO ZONAL C 85 . 1 LQ J  
  • 21. DEMANDA POBLACIONAL: Zona "A" Zona "B" Zona "C" Total Densid.prom.pob.(hab./edif.)RMD3-R3,R4 8.00 6.00 4.00 (Dato) Pob. total año 2045 s/modif.usos( hab.) 29,712 17,792 114,512 162,016 Dotac. 2045 s/Rgmto-OS100correg.(l /hab./día) 380.40 316.51 245.49 (Clima cálido) Cons. Prom. 2045 /modif. ( l/s) 130.82 65.18 325.37 521.36 100.0% CUADRO 01b.- DEMANDA CATASTRAL: Zona "A" Zona "B" Zona "C" Total Densidad prom.poblac.(hab./edif.) 8.00 6.00 4.00 Rptas:P4,P5,P6 Pob.RESID.Total de Saturación, Pd=?? ( hab.) 0 0 0 0 Rptas:P7,P8,P9 Pob.Tot.(RESID+FLOT.COM) Satur.Pd=??(hab.) 0 0 0 0 Dot.2019 s/Rgmto-os100(Lt./hab./día) 375.00 248.00 178.00 (Clima cálido) Dot. año catast. 2045 del Pd=20 (l/hab./día) 380.40 316.51 245.49 Qp.pob.RESID. 2045 c/mod. Pd=???(l/s) 128.96 51.07 235.92 415.94 Qp.pob.RESID.+FLOT.2045c/mod.Pd=20(l/s) 0.00 0.00 0.00 0.00 Para zonas comerciales e indutriales Qp.pob. año 2045 Final Pd=20 (l/s) 130.82 65.18 325.37 521.36 Qp catast.pob.sat.año 2045 Pd=20(l/s) ??? PÉRDIDAS EN LA DISTRIB. p/dato (Dependerá de gestión de optimiz.) 15.0% 599.56 600 l/s ii) Caudal ofertado por el sistema (l/s): 1,000.00 Caudal de demanda: Zona A+B+C(l/s): 600.00 1.66667 166.67% 0.60000 60.00% Reservorio existente el 2019 (Dato) 100 m3 GASTO CUADRO 02.- LOCALIDAD ß ( Consumo promedio año 2045) PROM. GASTO VOLUM. PRODUCC. VOLUM. DE PRODUCC. VOLUM. CONS. HORAS @ 2 horas PROM @ 2HR. VOLUMEN ACUMUL. ACUM.24Hr.2017 CONS.RESERV. AC.REAL RESERV.REAL % del Qprom. 2045 60.00% 60.00% 166.667% 166.667% (Lt./seg.) (Lts./s.) (Lts.) (Lts.) (Lts.) (Lts.) (Lts.) (Lts.) 0 0 0 0 1.00% 6.000 54,000 54,000 2,160,000 2,106,000 3,600,000 3,546,000 1 1.50% 9.000 157,680 211,680 4,320,000 4,108,320 7,200,000 6,988,320 2 5.80% 34.800 816,480 1,028,160 6,480,000 5,451,840 10,800,000 9,771,840 3 11.00% 66.000 626,400 1,654,560 8,640,000 6,985,440 14,400,000 12,745,440 Norma OS.030: Valmac. = Vregulac. + Vincendio + Vreserva Vregulac. = Diagrama Masa ó 25% del Qprom. en 24 horas Vreserva = 25% Vtotal Vreserva = 33% (Vregulac. + Vincendio) Vreserva = Qp x t ( 2 hr. < t > 4 hr. ) Vincendio = 50 m3 (para viviendas) ; GCIS con 3,000 m3 de volumen aparente (Edif. de uso comercial o industrial) Para determinar el caudal medio en Lt./seg. durante el año 2015, se procederá a desarrollar el cuadro que contiene las variaciones Factor de Deficit y Superavit : Efectuamos las estimaciones de las demanda por variaciones de consumo diarias en función a los datos suministrados del diario promedio horario y del diario maximo horario en Lt./seg., a fin de determinar los valores de K2.
  • 22. 4 12.00% 72.000 648,000 2,302,560 10,800,000 8,497,440 18,000,000 15,697,440 5 75.00% 450.000 2,095,200 4,397,760 12,960,000 8,562,240 21,600,000 17,202,240 6 98.00% 588.000 2,872,800 7,270,560 15,120,000 7,849,440 25,200,000 17,929,440 7 175.00% 1,050.000 4,968,000 12,238,560 17,280,000 5,041,440 28,800,000 16,561,440 8 210.00% 1,260.000 5,248,800 17,487,360 19,440,000 1,952,640 32,400,000 14,912,640 9 195.00% 1,170.000 4,255,200 21,742,560 21,600,000 -142,560 36,000,000 14,257,440 10 155.00% 930.000 3,386,880 25,129,440 23,760,000 -1,369,440 39,600,000 14,470,560 11 170.00% 1,020.000 3,693,600 28,823,040 25,920,000 -2,903,040 43,200,000 14,376,960 12 320.00% 1,920.000 8,538,134 37,361,174 28,080,000 -9,281,174 46,800,000 9,438,826 13 155.00% 930.000 3,352,680 40,713,854 30,240,000 -10,473,854 50,400,000 9,686,146 14 80.00% 480.000 2,419,200 43,133,054 32,400,000 -10,733,054 54,000,000 10,866,946 15 32.00% 192.000 1,080,000 44,213,054 34,560,000 -9,653,054 57,600,000 13,386,946 16 18.00% 108.000 777,600 44,990,654 36,720,000 -8,270,654 61,200,000 16,209,346 17 18.00% 108.000 864,000 45,854,654 38,880,000 -6,974,654 64,800,000 18,945,346 18 22.00% 132.000 1,231,200 47,085,854 41,040,000 -6,045,854 68,400,000 21,314,146 19 35.00% 210.000 1,944,000 49,029,854 43,200,000 -5,829,854 72,000,000 22,970,146 20 55.00% 330.000 1,900,800 50,930,654 45,360,000 -5,570,654 75,600,000 24,669,346 21 33.00% 198.000 756,000 51,686,654 47,520,000 -4,166,654 79,200,000 27,513,346 22 2.00% 12.000 82,080 51,768,734 49,680,000 -2,088,734 82,800,000 31,031,266 23 1.80% 10.800 60,480 51,829,214 51,840,000 10,786 86,400,000 34,570,786 24 1.00% 6.000 51,829,214 POR DÍA 451.70 51,829,214 51,829.21 600.00 19,295,294 1,000.00 34,570,786 k1= 1.30 (Reglam.) 19,295 34,571 M3 (Lts./s.) (Lts.) (M3) Q act-zona A+B Vol Rs (Qp.2018: A+B) (*) Dato 600 k2prom.19: 3.20 0.00 0.60 m3 PRE-DIMENSIONAMIENTO DEL RESERVORIO PRINCIPAL SIN USAR LOS COEFICIENTES DE VARIACIÓN HORARIA VRreserva= 25% Vralmacenam. Previa justificación. Se recomienda usar 25 % del Volumen total de almacenamiento VRalmacenam.= VRconsumo + VRincendio + VReserva = 12,957.3036 + 50.000 + 25% Vralmacenam. VRalmacenam.= (VRconsumo + VRincendio) / 0.75 = 17,343.071 m 3 17,343,071 litros Vreserva = 4,335.7679 m 3 VOLUMEN TOTAL RESERVORIO PRINCIPAL: 17,343.000 m 3 25.0% Rpta. G) 17,343 m3 DIFERENCIA DEL VOLUMEN DEL RESERV. REQUERIDO s/QOFERTA. Y s/ QDEMANDA 15,275.49 79.2% Para determinar los parámetros máximos de las variaciones horarias de consumo, se desarrollará el cuadro que contiene los datos en el día 14000000 CURVAS DE DEMANDA Y OFERTA DEL SISTEMA EN EL AÑO BASE DE OBSERVACIÓN: 2018
  • 23. GASTO CUADRO 03.- LOCALIDAD ß(Consumo del día máximo año 2045) PROM. GASTO VOLUM. PRODUCC. VOLUM. DE HORAS @ 1 hora PROM @ 2HR. VOLUMEN ACUMUL. ACUM.24 Hr. CONSUM.RES. % del Qprom. 100 % Dprom. 100% Dprom. (Lt./seg.) (Lts./s.) (Lts.) (Lts.) (Lts.) (Lts.) 0 45.00% 0.00 0 0 0 0 2 55.00% 0.00 0 0 0 0 4 70.00% 0.00 0 0 0 0 6 75.00% 0.00 0 0 0 0 8 125.00% 0.00 0 0 0 0 10 130.00% 0.00 0 0 0 0 12 150.00% 0.00 0 0 0 0 14 140.00% 0.00 0 0 0 0 16 115.00% 0.00 0 0 0 0 18 120.00% 0.00 0 0 0 0 20 155.00% 0.00 0 0 0 0 22 120.00% 0.00 0 0 0 0 24 45.00% 0.00 0 0 0 POR DÍA 0.00 0 0.00 Reserv(187.05 l/s)= 0 0 2000000 4000000 6000000 8000000 10000000 12000000 0 2 4 6 8 10 12 14 16 18 20 22 PRODUCC. AC.REAL 1.192974705 (Lts.) VOLUM. ACUMUL. 0.838240741 PRODUCC. AC.REAL 1.192974705 (Lts.) VOLUM. ACUMUL. 2035 (Lts.) PRODUCC. ACUM.24Hr.2035 0.838240741 (Lts.)
  • 24. (Lts./s.) (Lts.) (M3) 0 m3 0.00% 0.00 k2dis.(2018): 0.00 (Dìa de Consumo Máximo) k1= 1.30 (Reglam.) k2dis.2019: 3.20 (Dìa de Consumo Promedio) 2 Ptos. El año: 2025 El año: 2045 Único sistema de suministro: BOMBEO Dotación RNE-OS.100 : 220 l/hab/d Qm= ###### l/s Población 162,016 habitantes CIUDAD BETA Dotación (Catastro) : Dotac. VARIABLE Pob. A,B y C Qm= ###### l/s factor: 1 Caudales de Diseño: +15% PÉRDIDAS EN LA RED Qm= ###### l/s K1= 1.273 Qd1= 0.000 l/s K2= 2.401 Qd2= 0.000 l/s 781.46 DATOS PROPUESTO SIMILARES EN EL CUESTIONARIO-FICA 1439.34 Horas Demanda parcial (m3 ) 00:00 - 01:00 54.00000 01:00 - 02:00 157.68000 02:00 - 03:00 816.48000 03:00 - 04:00 626.40000 04:00 - 05:00 648.00000 05:00 - 06:00 2,095.20000 06:00 - 07:00 2,872.80000 07:00 - 08:00 4,968.00000 08:00 - 09:00 5,248.80000 09:00 - 10:00 4,255.20000 10:00 - 11:00 3,386.88000 11:00 - 12:00 3,693.60000 12:00 - 13:00 8,538.13440 13:00 - 14:00 3,352.68000 14:00 - 15:00 2,419.20000 15:00 - 16:00 1,080.00000 16:00 - 17:00 777.60000 17:00 - 18:00 864.00000 18:00 - 19:00 1,231.20000 19:00 - 20:00 1,944.00000 20:00 - 21:00 1,900.80000 21:00 - 22:00 756.00000 22:00 - 23:00 82.08000 El valor de k1 sera mayor y k2 menor de efectuarse evaluaciones de campo. Los coeficientes de consumo horario con respecto al promedio serán menores con tendencia central (picos menores) El valor de k1 sera mayor y k2 menor de efectuarse evaluaciones de campo. Los coeficientes de consumo horario con respecto al promedio serán menores con tendencia central (picos menores)
  • 25. 23:00 - 24:00 60.48000 ESCENARIO DE SISTEMA DE DISTRIBUCIÓN DE AGUA 2: BOMBEO- con 2 sub-escenarios Para efecto de comparación se inicia el bombeo de caudales a las 06:00 horas: Horas Demanda parcial (m3 ) Demanda acumulada (m3 ) Tiempo parcial (hora) Tiempo acumulado (hora) Caudal de bombeo parcial - 24horas- 3bombas (m3 /hora) Caudal de bombeo acumulado- 24horas- 3bombas (m3 /hora) Volumen del reservorio Bombeo 24horas- 3bombas (m3 ) Caudal de bombeo parcial - 8horas- 1bomba>capac. (m3 /hora) Caudal de bombeo acumulado- 8horas- 1bomba>capac. (m3 /hora) Volumen del reservorio Bombeo 8 horas - 1 bomba de > capac. (m3 ) 0.00000 0.00000 0.00000 0.00000 0.00000 06:00 - 07:00 2872.80000 2872.80000 1.00000 1.00000 2159.55060 2159.55060 -713.24940 6478.65180 6478.65180 3605.85180 07:00 - 08:00 4968.00000 7840.80000 1.00000 2.00000 2159.55060 4319.10120 -3521.69880 6478.65180 12957.30360 5116.50360 08:00 - 09:00 5248.80000 13089.60000 1.00000 3.00000 2159.55060 6478.65180 -6610.94820 6478.65180 19435.95540 6346.35540 09:00 - 10:00 4255.20000 17344.80000 1.00000 4.00000 2159.55060 8638.20240 -8706.59760 6478.65180 25914.60720 8569.80720 10:00 - 11:00 3386.88000 20731.68000 1.00000 5.00000 2159.55060 10797.75300 -9933.92700 25914.60720 5182.92720 11:00 - 12:00 3693.60000 24425.28000 1.00000 6.00000 2159.55060 12957.30360 -11467.97640 25914.60720 1489.32720 12:00 - 13:00 8538.13440 32963.41440 1.00000 7.00000 2159.55060 15116.85420 -17846.56020 25914.60720 -7048.80720 13:00 - 14:00 3352.68000 36316.09440 1.00000 8.00000 2159.55060 17276.40480 -19039.68960 25914.60720 -10401.48720 14:00 - 15:00 2419.20000 38735.29440 1.00000 9.00000 2159.55060 19435.95540 -19299.33900 6478.65180 32393.25900 -6342.03540 15:00 - 16:00 1080.00000 39815.29440 1.00000 10.00000 2159.55060 21595.50600 -18219.78840 6478.65180 38871.91080 -943.38360 16:00 - 17:00 777.60000 40592.89440 1.00000 11.00000 2159.55060 23755.05660 -16837.83780 6478.65180 45350.56260 4757.66820 17:00 - 18:00 864.00000 41456.89440 1.00000 12.00000 2159.55060 25914.60720 -15542.28720 6478.65180 51829.21440 10372.32000 18:00 - 19:00 1231.20000 42688.09440 1.00000 13.00000 2159.55060 28074.15780 -14613.93660 51829.21440 9141.12000 19:00 - 20:00 1944.00000 44632.09440 1.00000 14.00000 2159.55060 30233.70840 -14398.38600 51829.21440 7197.12000 20:00 - 21:00 1900.80000 46532.89440 1.00000 15.00000 2159.55060 32393.25900 -14139.63540 51829.21440 5296.32000 21:00 - 22:00 756.00000 47288.89440 1.00000 16.00000 2159.55060 34552.80960 -12736.08480 51829.21440 4540.32000 22:00 - 23:00 82.08000 47370.97440 1.00000 17.00000 2159.55060 36712.36020 -10658.61420 51829.21440 4458.24000 23:00 - 24:00 60.48000 47431.45440 1.00000 18.00000 2159.55060 38871.91080 -8559.54360 51829.21440 4397.76000 00:00 - 01:00 54.00000 47485.45440 1.00000 19.00000 2159.55060 41031.46140 -6453.99300 51829.21440 4343.76000 01:00 - 02:00 157.68000 47643.13440 1.00000 20.00000 2159.55060 43191.01200 -4452.12240 51829.21440 4186.08000 02:00 - 03:00 816.48000 48459.61440 1.00000 21.00000 2159.55060 45350.56260 -3109.05180 51829.21440 3369.60000 03:00 - 04:00 626.40000 49086.01440 1.00000 22.00000 2159.55060 47510.11320 -1575.90120 51829.21440 2743.20000 04:00 - 05:00 648.00000 49734.01440 1.00000 23.00000 2159.55060 49669.66380 -64.35060 51829.21440 2095.20000 05:00 - 06:00 2095.20000 51829.21440 1.00000 24.00000 2159.55060 51829.21440 0.00000 51829.21440 0.00000 90000 Título del gráfico
  • 27. PROCESO OPERATIVO-TÉCNICO-NORMATIVO PARA DETERMINAR EL VOLUMEN TOTAL DE RESERVORIOS EN POBLACIONES CON SOLO ACTIVIDADES DOMÉSTICAS Q3bomba= Demanda acumulada en 24 horas / 24 horas = 51,829.2144 ÷ 24.0000 = 2,159.551 m 3 /hora 599.875 l/s VRconsumo= 0.0000 - -19,299.3390 = 19,299.3390 m 3 19299339 litros Horas de bombeo = 8 horas Q1bomba= Demanda acumulada en 24 horas / 24 horas = 51,829.2144 ÷ 8.0000 = 6,478.652 m 3 /hora 1799.626 l/s VRconsumo= 10,372.3200 - -10,401.4872 = 20,773.8072 m3 20773807.2 litros RESPUESTA 01: Capacidad de descarga de una (01) bomba al operar 8 horas discontinuas durante el día (de 06:00 a 10:00 horas y de 14:00-18:00 horas), para lograr cubrir el total del volumen requerido por la demanda promedio de consumo o regulación diario de ciudad BETA es: En caso de usar una (01) sola bomba por 8 horas al día de forma discontinua (4 horas cada etapa), la capacidad del reservorio para permitir a la bomba cubrir de forma continua las 24 horas con la descarga requerida por la demanda de consumo de la ciudad BETA Capacidad de descarga de tres (03) bombas simples, operando cada uno 8 horas cubriendo el suministro continuo de caudal las 24 horas del día para satisfacer el total del volumen requerido por la demanda promedio de consumo diario de la ciudad BETA En caso se usar tres (03) bombas simples operando 24 horas (8 horas c/u) la capacidad del reservorio para permitir a la bombas cubrir de forma continua las 24 horas con la descarga requerida por la demanda de consumo o regulación de la ciudad BETA es: -20000 -10000 0 10000 20000 06:00 - 07:00 07:00 - 08:00 08:00 - 09:00 09:00 - 10:00 10:00 - 11:00 11:00 - 12:00 12:00 - 13:00 13:00 - 14:00 14:00 - 15:00 15:00 - 16:00 16:00 - 17:00 17:00 - 18:00 18:00 - 19:00 19:00 - 20:00 20:00 - 21:00 21:00 - 22:00 22:00 - 23:00 23:00 - 24:00 00:00 - 01:00 01:00 - 02:00 02:00 - 03:00 03:00 - 04:00 04:00 - 05:00 05:00 - 06:00 m 3 Horas Volumen del reservorio Bombeo 24horas-3bombas (m3) Volumen del reservorio Bombeo 8 horas -1 bomba de > capac. (m3)
  • 28. DIFER. DE VOLUMEN DE CONSUMO ESCENARIO (01) - (02) -1,474.4682 m 3 -1,474,468.20 litros VRconsumo= 0.0000 m3 VRconsumo= 0.0000 NO EXISTE VOLUMEN DE AGUA 0 m3 DE 14:00 -06:00 horas NO HABRÁ AGUA EN EL RESERVORIO RESPUESTA 02: DIECIOCHO (18) HORAS DIFER. DE VOLUMEN DE CONSUMO ESCENARIO (01) - (02) 0.0000 m3 VRconsumo= 0.0000 m 3 05:00 - 06:00 01 HORA DE MAYOR RIESGO VRconsumo= VRregulación= OS.030 RNE VRconsumo= 19,299.3390 m 3 19,295 m 3 RECALCULAR o VRconsumo= 12,957.3036 m 3 Para poblaciones >10,000 hab., netamente destinada a viviendas (OS.100.RNE) NO SE CONSIDERA OBLIGATORIO (Inciso A Art. 1.6 - Norma OS.100) VRincendio= 50 m3 (OS.030) VRincendio= 0 m3 (OS.100) Qi = 15 l/s 01 Grifo C.I.= 16 l/s VRincendio= 216.000 m3 Horas = 2 horas = 7200 segundos 4 horas = 14400 segundos VRreserva= 25% Vralmacenam. Previa justificación. Se recomienda usar 25 % del Volumen total de almacenamiento Optamos por el proceso que nos suministre un mayor valor resultante (En caso de existir compatibilidad con Vconsumo) (Usando el valor mín. si 50 m3 no es concordante con el Vcons.) De optar por un bombeo con tres (03) bombas funcionando 8 horas cada uno para cubrir de forma continua las 24 horas del día, el volumen que contiene el reservorio que satisfaga la demanda de consumo a las 06:00 horas es: De optar por un bombeo con una sola bomba por 8 horas discontinuas (De 06:00 a 10:00horas y de 14:00 a 18:00horas), en que hora ocurre el riesgo mayor de que el reservorio no tiene agua para cubrir la demanda de consumo por variaciones horarias, incendios y reserva para atender interrupciones, si se omite De elegir un sistema con tres (03) bombas funcionando 8 horas cada una para cubrir 24 horas de suministro continuo, el volumen de diseño del reservorio para cubrir a demanda de consumo por variaciones horarias, incendios y reserva para atender interrupciones, EN UNA ZONA NETAMENTE DESTINADA A VIVIENDA es: SE NECESITA UN RESERVORIO DE MAYOR VOLUMEN SE REQUIERE ÁNALISIS DE RENTABILIDAD AL CONTAR CON UN VOLUMEN PERMANENTE DE AGUA PARA CUBRIR VARIACIONES HORARIAS DE CONSUMO De optar por un bombeo con una sola bomba por 8 horas discontinuas, el volumen que contiene el reservorio para cubrir la demanda de consumo a las 06:00 horas es: Norma OS.030: Valmac. = Vregulac. + Vincendio + Vreserva Vregulac. = Diagrama Masa ó 25% del Qprom. en 24 horas Vreserva = 25% Vtotal Vreserva = 33% (Vregulac. + Vincendio) Vreserva = Qp x t ( 2 hr. < t > 4 hr. ) Vincendio = 50 m3 (para viviendas) ; GCIS con 3,000 m3 de volumen aparente (Edif. de uso comercial o industrial)
  • 29. VRalmacenam.= VRconsumo + VRincendio + VReserva = 19,299.3390 + 0.000 + 25% Vralmacenam. VRalmacenam.= (VRconsumo + VRincendio) / 0.75 = 25,732.452 m 3 25,732,452 litros Vreserva = 6,433.1130 m 3 VOLUMEN TOTAL RESERV-ESCENARIO 01: 03 BOMBAS/24HR: 25,732.000 m 3 25.0% VRconsumo= 20,773.8072 m 3 VRconsumo= VRregulación= OS.030 RNE Para poblaciones >10,000 hab., netamente destinada a viviendas (OS.100.RNE) NO SE CONSIDERA OBLIGATORIO (Inciso A Art. 1.6 - Norma OS.100) VRincendio= 50 m3 (OS.030) VRincendio= 0 m3 (OS.100) VRincendio= 0.000 m 3 Qi = 15 l/s 1 hidrante de 15 l/s cada uno Horas = 2 horas = 7200 segundos 4 horas = 14400 segundos VRreserva= 25% Vralmacenam. Previa justificación. Se recomienda usar 25 % del Volumen total de almacenamiento VRalmacenam.= VRconsumo + VRincendio + VReserva = 20,773.8072 + 0.000 + 25% Vralmacenam. VRalmacenam.= (VRconsumo + VRincendio) / 0.75 = 27,698.410 m 3 27,698,410 litros Vreserva = 6,924.6024 m 3 VOLUMEN TOTAL RESERV.-ESCENAR.02- 01 BOMBAS/08 HR: 27,698.000 m 3 25.0% DIFERENCIA: 1,965.958 m 3 1,965,958 litros 1,965.958 m 3 PROCESO OPERATIVO-TÉCNICO-NORMATIVO PARA DETERMINAR EL VOLUMEN TOTAL DE RESERVORIOS EN POBLACIONES CON SOLO ACTIVIDADES DOMÉSTICAS VRconsumo= 19,299.3390 m 3 VRconsumo= VRregulación= OS.030 RNE Usando el gráfico para agua contra incendio de sólidos y las recomendaciones mínimas establecidas en la Norma OS.030 RNE: Optamos por el proceso que nos suministre un mayor valor resultante (En caso de existir compatibilidad con Vconsumo) (Usando el valor mín. si 50 m3 no es concordante con el Vcons.) De optar por un bombeo con una (01) bomba funcionando 8 horas discontinuas de 4 horas (de 06:00 a 10:00 horas y de 14:00 a 18:00 horas) cada etapa, el De optar por un bombeo con tres (03) bombas funcionando 8 horas cada una para cubrir 24 horas de suministro continuo, el volumen de diseño del
  • 30. Volumen aparente de incendio mínimo = 3,000 m3 Factor de apilamiento intermedio (g)= 0.1 Para determinar el Volumen del reservorio contra incendio, del gráfico se obtiene:
  • 31. Qincendio= 48 l/s Para extinguir el fuego Qi = 48 l/s 3 hidrantes de 16 l/s cada uno Horas = 2 horas = 7200 segundos 4 horas = 14400 segundos VRincendio= 345.600 m3 (Usando el promedio Vol. Aparente y g=0.1 y tmín. 2 horas) Para determinar el Volumen del reservorio para reserva, del gráfico se obtiene: VRreserva= 145 m3 VRalmacenam.= VRconsumo + VRincendio + VReserva = 19,299.3390 + 345.600 + 145 VRalmacenam.= 19,789.939 m3 19,789,939 litros DIFERENCIA: -5,942.513 5,942,513 litros 5,942.513 m 3 VRconsumo= 20,773.8072 m3 VRconsumo= VRregulación= OS.030 RNE Usando el gráfico para agua contra incendio de sólidos y las recomendaciones mínimas establecidas en la Norma OS.030 RNE: Volumen aparente de incendio mínimo = 3,000 m3 Factor de apilamiento intermedio (g)= 0.5 Para determinar el Volumen del reservorio contra incendio, del gráfico se obtiene: Qincendio= 48 l/s Para extinguir el fuego Qi = 48 l/s 4 hidrantes de 16 l/s cada uno Horas = 2 horas = 7200 segundos 4 horas = 14400 segundos VRincendio= 345.600 m3 (Usando el promedio Vol. Aparente y g=0.1 y tmín. 2 horas) Para determinar el Volumen del reservorio para reserva, del gráfico se obtiene: VRreserva= 145 m3 VRalmacenam.= VRconsumo + VRincendio + VReserva = 20,773.8072 + 345.600 + 145 VRalmacenam.= 21,264.407 m3 21,264,407 litros RESPUESTA 03: DIFERENCIA: 1,474.468 m3 1,474,468 litros 1,474.468 m 3 DIFERENCIAS ENTRE ZONA DE VIVIEND. DE COMERC.INDUST. -6,434.002 m3 De optar por un bombeo con una (01) bomba funcionando 8 horas discontinuas de 4 horas cada etapa (de 06:00 a 10:00 horas y de 14:00 a 18:00 horas), el